SSC GD 2015 Previous Year Paper

SSC GD 2015 

Reasoning 

Instructions 

For the following questions answer them individually 

Q. 1 If the alphabets are numbers the sum of which 5 alphabets is 51. 

(A) AEOIT 

(B) AIOEJ 

(C) AOUEH 

(D) AIOEU 

Answer: (D) 

Explanation: 

(A) : AEOIT ; 1 + 5 + 15 + 9 + 20 = 50 

(B) : AIOEJ ; 1 + 9 + 15 + 5 + 10 = 40 

(C) : AOUEH ; 1 + 15 + 21 + 5 + 8 = 50 

(D) : AIOEU ; ; 1 + 9 + 15 + 5 + 21 = 51 

=> Ans – (D) 

Instructions 

Find the odd word/letter/number from the given alternatives. 

 

Q. 2 

(A) YVX 

(B) QNO 

(C) EBD

(D) IFH 

Answer: (B) 

Explanation: 

(A) : Y (-3) = V (+2) = X 

(B) : Q (-3) = N ( +1) = O 

(C) : E (-3) = B (+2) = D 

(D) : I (-3) = F (+2) = H 

=> Ans – (B) 

 

Q. 3 

(A) 424 

(B) 631 

(C) 432 

(D) 460 

Answer: (C) 

Explanation: 

Except the third option, sum of digits of rest of the numbers is same. 

(A) : 424 ; 4 + 2 + 4 = 10 

(B) : 631 ; 6 + 3 + 1 = 10 

(C) : 432 ; 4 + 3 + 2 =

(D) : 460 ; 4 + 6 + 0 = 10 

=> Ans – (C) 

 

Q. 4 

(A) Triangle 

(B) Cube 

(C) Square 

(D) Trapezium 

Answer: (B) 

Explanation: 

Triangle, square and trapezium are two dimensional figures, hence cube being a 3-D figure is the odd one. => Ans – (B) 

Instructions 

For the following questions answer them individually 

Q. 5 Which figure represents the relation amongst rose, flower and leaves. 

Answer: (A) 

Explanation: 

Roses comes under the category of flowers, but leaves are different, hence the venn diagram that best describes above relation is :

=> Ans – (A) 

 

Q. 6 Raghu starts from his house in his car and travels 8 km towards the North, then 6 km towards East, then 10 km towards his right, 4 km towards his left, 10 km toward north and finally 4 km towards his right. In which direction is he now with reference to the starting point ? 

(A) North 

(B) South-East 

(C) South 

(D) North-East 

Answer: (D) 

Explanation: 

Raghu starts from his house in his car at (A) and travels 8 km towards the North to B, then 6 km towards East to reach C, then 10 km towards his right towards south, 4 km towards his left in the east direction, 10 km toward north to reach F and finally 4 km towards his right to stop at point G. 

∴ He is now in North-East direction with reference to the starting point. 

=> Ans – (D) 

Instructions 

A series is given, with one/two terms missing. Choose the correct alternative from the given ones that will complete the series. 

 

Q. 7 A, Z, B, Y, C, X, D, W, E, V, F, U, G, ? 

(A)

(B)

(C)

(D)

Answer: (A) 

Explanation: 

Above series is a combination of two alternate series, one being in increasing order and other in reverse order according to the English alphabetical series. 

1st : A, B, C, D, E, F, G 

2nd : Z, Y, X, W, V, U, T 

=> Ans – (A) 

 

Q. 8 3, 8, 10, 15, 17, 22, 24, ?, ? 

(A) 26, 28 

(B) 29, 31 

(C) 29, 32 

(D) 29, 34 

Answer: (B) 

Explanation: 

The numbers ‘5’ and ‘2’ are alternatively added. 

3 + 5 = 8 

8 + 2 = 10 

10 + 5 = 15 

15 + 2 = 17 

17 + 5 = 22 

22 + 2 = 24 

24 + 5 = 29 

29 + 2 = 31 

=> Ans – (B) 

Instructions 

For the following questions answer them individually 

Q. 9 Which of the answer figures can be formed using the questions figures. 

Answer: (A) 

 

Q. 10 In a certain code TEMPLE is written as METELP. How is FAITHFUL written in that code ? 

(A) TIAFLUFH 

(B) TAIFULFH 

(C) TAFILUFH 

(D) TIAFFULH 

Answer: (A) 

Explanation: 

TEMPLE is written as METELP 

If we break the word in half, i.e. ‘TEM’ and ‘PLE’ and reverse each of the part, we get the desired result.

Eg :- TEM -> MET and PLE -> ELP 

Similarly, FAIT -> TIAF and HFUL -> LUFH 

Thus, FAITHFUL : TIAFLUFH 

=> Ans – (A) 

Instructions 

Select the related word/letter/ number from the given alternatives. 

Q. 11 CARD: JOKER : : BOOK : ? 

(A) WORDS 

(B) WRITER 

(C) PAGES 

(D) COVER 

Answer: (B) 

 

Q. 12 8 : 81 : : 64 : ? 

(A) 525 

(B) 625 

C125 

(D) 137 

Answer: (B) 

 

Q. 13 AK : FP : : XD : ? 

(A) SJ 

(B) CI 

(C) BH 

(D) TE 

Answer: (B) 

Explanation: 

Expression = AK : FP : : XD : ? 

The pattern followed is : A (+6) = F and K (+6) = P 

Similarly, X (+6) = C and D (+6) = I 

Thus, XD : CI 

=> Ans – (B) 

 

Q. 14 WALLET : MONEY : : ENVELOPE : ? 

(A) GUM 

(B) POSTOFFICE 

(C) SUITCASE 

(D) LETTER 

Answer: (D) 

Explanation: 

Second is kept in first, money is kept in wallet, similarly an envelope contains a letter.

=> Ans – (D) 

Instructions 

For the following questions answer them individually 

Q. 15 A word is represented by only one set of numbers as given in any one of the alternatives. The sets of numbers given in the alternatives are represented by two classes of alphabets as in two Matrices given below. The columns and rows of Matrix-I are numbered from 0 to 4 and that of Matrix-II are numbered from 5 to 9. A letter from these matrices can be represented first by its row and next by its column, e.g., “T” can be represented by 03, 12 etc., and ‘M’ can be represented by 55, 67 etc. Similarly, you have to identify the set for the word ‘RUDE’. 

(A) 59, 99, 34, 11 

(B) 77, 56, 02, 01 

(C) 95, 87, 42, 12 

(D) 56, 65, 10, 33 

Answer: (A) 

Explanation: 

(A) : 59, 99, 34, 11 : RUDE 

(B) : 77, 56, 02, 01 : RUED 

(C) : 95, 87, 42, 12 : RUDT 

(D) : 56, 65, 10, 33 : UUDE 

=> Ans – (A) 

Q. 16 Which answer figure will complete the pattern in the Question figure ? 

Answer: (A) 

Explanation: 

In each row, (if we look at the small 2-(D) figures), each row contains a square, triangle and circle, thus in the last row, the missing figure will definitely contain a circle, hence the middle two options are eliminated. 

Also, in each row, the two semi circles are in opposite directions, thus first is the correct option. 

=> Ans – (A) 

 

Q. 17 From the given alternative words, select the word which cannot be formed using the letters of the given word. 

ADMINISTRATION 

(A) SITUATION 

(B) RATION 

(C) STRAIN 

(D) TRADITION 

Answer: (A) 

Explanation: 

The word ADMINISTRATION does not contain any ‘U’, thus the term Situation cannot be formed. 

=> Ans – (A) 

Instructions 

Two statements are given followed by two Conclusions / Assumptions, I and II. You have to consider the statement to be true, even if it seems at variance from commonly known facts. You are to decide which of the given conclusions / assumptions can definitely be drawn from given statements. Indicate your answer. 

 

Q. 18 Statements 

1. No teacher come to the school on a bicycle. 

b: Anand comes to the school on a bicycle. 

Conclusions 

I. Anand is not a teacher 

II. Anand is a student. 

(A) Conclusion II alone can be drawn. 

(B) Both conclusions cannot be drawn. 

(C) Both conclusions can be drawn. 

(D) Conclusion I alone can be drawn. 

Answer: (D) 

Explanation: 

The given statement indicates that none of the teachers of a school uses bicycle to come to the school and Anand comes to school using a bicycle, thus this concludes that Anand is not a teacher, but we cannot know whether Anand is a student or not. 

Thus, conclusion I alone can be drawn. 

=> Ans – (D) 

Instructions 

For the following questions answer them individually 

Q. 19 The number of students in an art class is increasing month after month as follows. Find the number of students in June from the following information. 

(A) 16 

(B) 13 

(C) 15 

(D) 14 

Answer: (A) 

Explanation: 

Consecutive natural numbers are added. 

1 + 1 = 2 

2 + 2 = 4 

4 + 3 = 7 

7 + 4 = 11 

11 + 5 = 16 

=> Ans – (A) 

 

Q. 20 In the question, if a mirror is placed on the line AB then which of the answer figures is the right image of the given figure ? 

Answer: (B) 

Explanation: 

A vertical mirror is placed, so the object on the left will appear right in reverse position and vice-versa. So all the arrows will point in the same direction alternatively, thus the first and last options will be eliminated. 

Also, in the Q. figure, the first arrow is at the top which will still stay at the top but will face leftwards, hence second option is the right image. 

=> Ans – (B) 

 

Q. 21 A piece of paper is folded and cut as shown below in the Question figures. From the given answer figures, indicate how it will appear when opened ? 

Answer: (B) 

 

Q. 22 From the given answer figures, select the one in which the Question figure is hidden / embedded. 

Answer: (D) 

 

Q. 23 Which will appear 3rd in the dictionary ? 

(A) collision 

(B) collegiate 

(C) collinear 

(D) colloquy 

Answer: (A) 

Explanation: 

As per the order of dictionary, 

= collegiate -> collinear -> collision -> colloquy 

Thus, 3rd word is collision 

=> Ans – (A) 

 

Q. 24 Find the missing number from the given responses ? 

(A)

(B)

(C)

(D)

Answer: (D) 

Explanation: 

The number at the top is subtracted from the sum of bottom two numbers to get the middle number. 

Eg :- (8 + 4) − 3 = 9 

and 

(6 + 5) − 4 = 7 

Similarly, 

(9 + 3) − 5 = 7 

=> Ans – (D) 

 

Q. 25 If 50 minutes ago, it was 45 minutes past four O clock, how many minutes is it until six ‘O clock ? 

(A) 45 

(B) 15 

(C) 25 

(D) 35 

Answer: (C) 

Explanation: 

45 minutes past four O clock = 4:45 

Thus, present time = 4:45 + 50 minutes = 5:35 

Thus minutes until six ‘o clock = 6:00 – 5:35 = 25 minutes 

=> Ans – (C) 

Quant 

Instructions 

For the following questions answer them individually 

Q. 26 A 200 metre long train is running at a speed of 72 km/ hr How long will it take to cross 800 meter long bridge ? 

(A) 50 seconds 

(B) 40 seconds 

(C) 60 seconds 

(D) 30 seconds 

Answer: (A) 

 

Q. 27 The average age of a husband and is wife was 23 years at the time of their marriage. After five years they have a one year old child. The average age of the family now is 

(A) 29.3 years 

(B) 19 years 

(C) 23 years 

(D) 28.5 years 

Answer: (B) 

 

Q. 28 A use worth Rs.1,50,000 is sold by X to Y at 5% profit. Y sells the house back to X at 2% loss. Then in the entire transaction: 

(A) X gains Rs.3150 

(B) X loses Rs.4350 

(C) X loses Rs.1350 

(D) X gains Rs.4350 

Answer: (A) 

 

Q. 29 The value of (75.8)2 −(35.8)2 /40 is 

(A) 121.6 

(B) 40 

(C) 160 

(D) 111.6 

Answer: (D) 

 

Q. 30 What should be the least number of years in which the simple interest on Rs.2600 at 6⅔% will be an exact number of rupees ? 

(A)

(B)

(C)

(D)

Answer: (A) 

 

Q. 31 A man allows a discount of 10% on a book whose marked price is Rs.40. What is the cost price so that the profit is 20% ? 

(A) Rs.35 

(B) Rs.40 

(C) Rs.30 

(D) Rs.45 

Answer: (C) 

 

Q. 32 The three angles of a triangle are in the ratio 3 : 4 : 5. Then the angles respectively are: 

(A) 45, 60, 75 

(B) 60, 45, 75 

(C) 60, 75, 45 

(D) 75, 60, 45 

Answer: (A) 

 

Q. 33 The amount of rice produced in a square field of side 50 m is 750 kg. The amount of rice produced in a similar square field of side 100 m will be 

(A) 2000 kg 

(B) 3000 kg 

(C) 3500 kg 

(D) 1500 kg 

Answer: (B) 

 

Q. 34 The sum of all natural numbers from 75 to 97 is: 

(A) 1598 

(B) 1978 

(C) 1798 

(D) 1958 

Answer: (B) 

 

Q. 35 Six friends have an average height of 167 cms. A boy with height 162 cm leaves the group. Find the new average height. 

(A) 168 cm 

(B) 166 cm 

(C) 169 cm 

(D) 167 cm 

Answer: (A) 

 

Q. 36 If x, y are rational numbers and 5 + 113 – 211= x + y 11. The values of x and y are 

(A)  X= -14/17 ,Y= -13/26

(B)  X= 4/13 , Y= 11/17

(C)  X= -27/25 ,Y= -11/37

(D)  X= -37/35 ,Y= -13/35

Answer: (D) 

 

Q. 37 If the radius of a circle is decreased by 10% then the area of the circle is decreased by 

(A) 89% 

(B) 18% 

(C) 19% 

(D) 25% 

Answer: (C) 

 

Q. 38 Ritu purchased 2½ dozen eggs at the rate of Rs.20 per dozen. She found that 6 eggs were rotten. She sold the remaining eggs at the rate of Rs.22 per dozen. Then her profit or loss percent is: 

(A) 12% loss 

(B) 12% profit 

(C) 10% loss 

(D) 10% profit 

Answer: (A) 

Instructions 

The following pie-chart shows the market share of four companies S, T, U and V. Total market is worth Rs.72 crores. Study the pie-chart and answer the questions. 

Q. 39 The company having maximum market share is 

(A)

(B)

(C)

(D)

Answer: (D) 

 

Q. 40 The difference of market shares of companies V and U is 

(A) Rs.8 crores 

(B) Rs.9 crores 

(C) Rs.6 crores 

(D) Rs.4 crores 

Answer: (D) 

Instructions 

For the following questions answer them individually 

 

Q. 41 The time required for a boy to travel along the external and internal boundaries of a circular path are in the ratio 20: 19. If the width of the path be 5 meters, the internal diameter is: 

(A) 195 metres 

(B) 192 metres 

(C) 180 metres 

(D) 190 metres 

Answer: (D) 

 

Q. 42 8 children and 12 men complete a certain piece of work in 9 days, Each child takes twice the time taken by a man to finish the work. In how many days will 12 men finish the same work ? 

(A) 9 days 

(B) 13 days 

(C) 12 days 

(D) 15 days 

Answer: (C) 

 

Q. 43 A certain number of men can do a work in 40 days. If there we 8 men more, it could be finished in 10 days less. How many men were there initially ? 

(A) 20 

(B) 24 

(C) 30 

(D) 16 

Answer: (B) 

 

Q. 44 If Rs.510 be divided among A,B and C in such a way that A gets 2/3 of what B gets and B gets 1/4 of what C gets, then their shares are respectively. 

(A) Rs.150, Rs.240, Rs.120 

(B) Rs.60, Rs.90, Rs.360 

(C) Rs.120, Rs.240, Rs.150 

(D) Rs.150, Rs.300, Rs.60 

Answer: (B) 

 

Q. 45 The average weight of 8 persons increases by 2.5 kg when a new persons comes in place of one of them weighing 65 kg. The weight of the new person is 

(A) 84 kg 

(B) 85 kg 

(C) 76 kg 

(D) 76.5 kg 

Answer: (B) 

 

Q. 46 The sum of two positive numbers is 20% of the sum of their squares and 25% of the difference of their squares. If the numbers are x and y the, x + y/x3 is equal to 

(A) 1/4

(B) 3/8

(C) 1/3

(D) 2/9 

Answer: (D) 

 

Q. 47 A seller gains 20% profit even after allowing 10% discount. If the amount of profit on a TV set is Rs.750, then the marked price of the TV set is 

(A) Rs.5200 

(B) Rs.5000 

(C) Rs.4800 

(D) Rs.5500 

Answer: (B) 

 

Q. 48 Ram bought a bike for Rs.60,000. He paid Rs.10,000 cash down and the rest at the end of 2 years at 15% simple interest. How much more did he pay as simple interest ? 

(A) Rs.15,000 

(B) Rs.25,000 

(C) Rs.35,000 

(D) Rs.50,000 

Answer: (A) 

 

Q. 49 A man rows 750 m in 675 seconds against the stream and returns in 7½ minutes. Its rowing speed in still water is (in km/hr). 

(A) 5.5 

(B) 5.75 

(C)

(D) 5.25 

Answer: (D) 

 

Q. 50 A scooter is sold at three successive discounts of 10%, 5% and 2%. If the marked price of the scooter is Rs.18,000, find its net selling price. 

(A) Rs.15028.20 

(B) Rs.15082.00 

(C) Rs.15082.20 

(D) Rs.15080.00 

Answer: (C) 

English 

Instructions 

In the following questions, some parts of the sentences have errors and some are correct. Find out which part of a sentence has an 

error. The number of that part is the answer. If a sentences is free from error, your answer is No Error. 

Q. 51 An idea was worth nothing if it has no champion. 

(A) No error 

(B) If it has no champion 

(C) An idea 

(D) was worth nothing 

Answer: (D) 

 

Q. 52 The camp beside ours has been built in 1966 by John’s brother. 

(A) No error 

(B) by John;s brother 

(C) The camp beside ours 

(D) has been built in 1966. 

Answer: (D) 

 

Q. 53 I have been waiting for you since two hours. 

(A) for you 

(B) No error 

(C) I have been waiting 

(D) since two hours 

Answer: (D) 

Instructions 

In the following questions, out of the four alternatives, choose the one which can be substituted for the given words / sentences. 

 

Q. 54 To put two and two together 

(A) Proud 

(B) Good friend 

(C) Selfish friend 

(D) Understand 

Answer: (D) 

 

Q. 55 A container for the ashes of a dead person 

(A) Vessel 

(B) Vase 

(C) Jug 

(D) Urn 

Answer: (D) 

 

Q. 56 One who pretends to be what he is not 

(A) Hypocrite 

(B) Turncoat 

(C) Liar 

(D) Actor 

Answer: (A) 

Instructions 

In the following questions, a part of the sentence is printed is bold. Below are given alternatives to the bold part which may improve the sentence. Choose the correct alternatives. In case no improvement is needed your answer is No improvement. 

Q. 57 Journalism and medicine would be two of his career options. 

(A) No improvement 

(B) could be 

(C) will be 

(D) might be 

Answer: (D) 

 

Q. 58 No economist can accurately foresee whether ta will go up or down. 

(A) expect 

(B) anicipate 

(C) No improvement 

(D) obviate 

Answer: (B) 

 

Q. 59 One should keep their word. 

(A) One’s 

(B) his 

(C) everyone’s 

(D) No improvement 

Answer: (A) 

Instructions 

In the following three questions, choose the word opposite in meaning to the given word. 

 

Q. 60 NOISILY 

(A) loudly 

(B) quietly 

(C) clearly 

(D) distinctly 

Answer: (B) 

 

Q. 61 ADMIRATION 

(A) blame 

(B) contempt 

(C) disapprove 

(D) despise 

Answer: (B) 

 

Q. 62 BEAUTIFUL 

(A) bountiful 

(B) unique 

(C) bizarre 

(D) ugly 

Answer: (D) 

Instructions 

In the following questions, four words are given in each question, out of which only one word is correctly spelt. Find the correctly spelt word as your answer. 

Q. 63 

(A) complacency 

(B) complacensy 

(C) cumplacency 

(D) complicency 

Answer: (A) 

 

Q. 64 

(A) compelsory 

(B) compullsory 

(C) compulsorry 

(D) compulsory 

Answer: (D) 

 

Q. 65 

(A) entusiasm 

(B) enthussiasm 

(C) enthusiasm 

(D) enthusaism 

Answer: (D) 

Instructions 

In the following three questions, out of the four alternatives, choose the one which best expresses the meaning of the given word. 

 

Q. 66 ENORMOUS 

(A) Petty 

(B) Warehouse 

(C) Immense 

(D) Trivial 

Answer: (C) 

 

Q. 67 INEVITABLE 

(A) Significant 

(B) Unavoidable 

(C) Crucial 

(D) Undentable 

Answer: (B) 

 

Q. 68 DRIZZLE 

(A) Sprinkle 

(B) Trickle 

(C) Splash 

(D) Downpour 

Answer: (A) 

Instructions 

In the following questions, sentences are given with blanks to ge filled in with an appropriate word (s). Four alternatives are suggested for each question. Choose the correct alternative out of the four as your answer. 

Q. 69 We get milk from the_______. 

(A) dairy 

(B) daily 

(C) daisy 

(D) diary 

Answer: (A) 

 

Q. 70 A man from our village has been nominated ____ the ruling party’s candidate for the post. 

(A) to 

(B) as 

(C) in 

(D) for 

Answer: (B) 

 

Q. 71 He has been staying in Delhi ____ a long time. 

(A) for 

(B) since 

(C) from 

(D) till 

Answer: (A) 

 

Q. 72 Jones is a member of our ______ 

(A) coup 

(B) council 

(C) counter 

(D) counsel 

Answer: (B) 

Instructions 

In each of the following questions, four alternatives are given for the Idiom / Phrase printed is bold in the sentence. Choose the alternatives which best expresses the meaning of the Idiom / Phrase as your answer. 

 

Q. 73 Television has become part and parcel of our lives. 

(A) status symbol 

(B) unavoidable luxury 

(C) important part 

(D) showy part 

Answer: (C) 

 

Q. 74 My kith and kin congratulated me on my brilliant success. 

(A) niece and nephew 

(B) father and mother 

(C) relatives 

(D) colleagues 

Answer: (C) 

 

Q. 75 His frequent journeys are telling upon his health. 

(A) threatening 

(B) improving 

(C) informing 

(D) affecting 

Answer: (D) 

General Awareness 

Instructions 

For the following questions answer them individually 

Q. 76 A person with blood group O’ can receive blood from a person with blood group____ 

(A) Only A, B 

(B) O & A B

(C) Only O 

(D) A, B & O 

Answer: (C) 

 

Q. 77 The first black president of of South Africa and anti apartheid activist Nelson Mandela passed away on ? 

(A) 2nd Dec 2013 

(B) 5th Dec 2013 

(C) 9th Dec 2014 

(D) 6th Dec 2014 

Answer: (B) 

 

Q. 78 What are the extremely hot and acid local winds blowing in North-Indian plains in summers called ? 

(A) Mistral 

(B) Jet Stream 

(C) Loo 

(D) Fin 

Answer: (C) 

 

Q. 79 What is that nuclear reaction called in which mass turns into energy ? 

(A) Endoergic 

(B) Endothermic 

(C) Exothermic 

(D) Exoergic 

Answer: (D) 

 

Q. 80 Ramakrishna mission was established by ? 

(A) Swamy Vivekananda 

(B) Eshwar Chandra Vidyasagar 

(C) Dayanand Saraswathi 

(D) Raja Ramohan Roy 

Answer: (A) 

 

Q. 81 Which of the following is called ‘Continent of contrasts’ ? 

(A) Africa 

(B) Asia 

(C) Australia 

(D) Antarctica 

Answer: (A) 

 

Q. 82 Who is the first prime minister who submitted his/her resignation letter outside the parliament session ? 

(A) Moraji Desai 

(B) Lal Bahadur Sastri 

(C) Chaudhary Charan Singh 

(D) Indira Gandhi 

Answer: (C) 

 

Q. 83 Which is the second largest continent in the world ? 

(A) North America 

(B) Africa 

(C) Asia 

(D) Antarctica 

Answer: (B) 

 

Q. 84 According to the constitution the public health, sanitation, hospitals and dispensaries comes under which list ? 

(A) Union List 

(B) Concurrent List 

(C) Does not come under any list 

(D) States List 

Answer: (D) 

 

Q. 85 Which of the following is not a central problem of economy ? 

(A) What to produce 

(B) For whom to produce 

(C) How to produce 

(D) How to maximize profit 

Answer: (D) 

 

Q. 86 Which Mauryan Emperor went to Shravanabelagola along with Bhadrabahu ? 

(A) Dasaratha 

(B) Bindusara 

(C) Ashoka 

(D) Chandragupta 

Answer: (D) 

 

Q. 87 Who presented the ‘Dynamic Theory of Profit’ ? 

(A) Howle 

(B) Clarke 

(C) Walker 

(D) Knight 

Answer: (B) 

 

Q. 88 What are those grey clouds called which causes rain? 

(A) cumulus 

(B) cirrus 

(C) nimbostratus 

(D) nimbus 

Answer: (C) 

 

Q. 89 Which of the following methods is not used to remove permanent hardness of water? 

(A) by mixing sodium carbonate 

(B) by distillation 

(C) by mixing caustic soda 

(D) by boiling 

Answer: (D) 

 

Q. 90 Who is the woman boxer to win silver medal in London Olympics of 2012? 

(A) Natasha Jonas 

(B) Mary Kom 

(C) Katie taylor 

(D) Nicola Adams 

Answer: (D) 

 

Q. 91 Who was the Governor-General of India at the time of Revolt of 1857? 

(A) Lytton 

(B) Minto 

(C) Canning 

(D) Dalhousie 

Answer: (C) 

 

Q. 92 The census is caused out once in how many years ? 

(A) 5 years 

(B) every year 

(C) 10 years 

(D) 15 years 

Answer: (C) 

 

Q. 93 What did Louis Pasteur discover ? 

(A) Polio vaccine 

(B) Insulin 

(C) Anti Rabies vaccine 

(D) Penicillin 

Answer: (C) 

 

Q. 94 Which was the movement undertaken by the Indian Government to increase milk production ? 

(A) Green Revolution 

(B) White Revolution 

(C) Blue Revolution 

(D) Yellow Revolution 

Answer: (B) 

 

Q. 95 Which of the following countries was the first to given women voting nights ? 

(A) New Zealand 

(B) India 

(C) Iceland 

(D) America 

Answer: (A) 

 

Q. 96 Which countries are part of the group collectively termed as the Baltic countries ? 

(A) Poland, Belarus and Lithuania 

(B) Estonia, Latvia and Lithuania 

(C) Denmark, Poland and Latvia 

(D) Sweden, Finland and Estonia 

Answer: (B) 

 

Q. 97 Which of the following is not an aerosol ? 

(A) Smoke 

(B) Mud 

(C) Fog 

(D) Cloud 

Answer: (B) 

 

Q. 98 The first Electronic digital computer had which of the following components ? 

(A) Electronic Valve 

(B) Semiconductor Memory 

(C) Vacuum tube 

(D) Transistor 

Answer: (C) 

 

Q. 99 Amphoteric substance reacts in which of the following ways ? 

(A) Base 

(B) Both Base & Acid 

(C) None of these 

(D) Acid 

Answer: (B) 

 

Q. 100 Which day is celebrated as ‘National Youth Day’ in India ? 

(A) 12th January 

(B) 20th January 

(C) 1st January 

(D) 30th January 

Answer: (A) 

SSC GD 2012 Shift-II Previous Year Paper

SSC GD 2012 Shift-II

Quant 

Instructions 

For the following questions answer them individually 

Q. 1 By how much is 3/5 th of 75 greater than 4/7th of 77? 

(A)

(B)

(C)

(D) None of these 

Answer: (C) 

 

Q. 2 The HCF of two numbers 24 and their LCM is 216. If one of the number is 72, then the other number is 

(A) 27 

(B) 72 

(C)

(D) 24 

Answer: (B) 

 

Q. 3 3/4 of 1/7 of a number 120, then the number 

(A) 1120 

(B) 560 

(C) 280 

(D) 140 

Answer: (A) 

 

Q. 4 In a co-educational secondary school 65% of the students are boys. If there are 224 girls in the school, find the number of boys in the school ? 

(A) 400 

(B) 425 

(C) 416 

(D) None of these 

Answer: (C) 

 

Q. 5 A mixture of 30 liters of spirit and water contains 20% of water in it. How much water must be added to it, to make the water 25% in the new mixture ? 

(A) 3 liters 

(B) 2 liters 

(C) 4 liters 

(D) None of these 

Answer: (B) 

 

Q. 6 The ages of x and y are in the ratio 3:1. Fifteen years hence, the ratio will be 2:1. The present ages (in years) are: 

(A) 30, 10 

(B) 45, 15 

(C) 21, 7 

(D) 60, 20 

Answer: (B) 

 

Q. 7 Gold is 19 times as heavy as water and copper is 9 times as heavy as water. In what ratio should these be mixed to get an alloy 15 times as heavy as water ? 

(A) 1 : 1 

(B) 2 : 1 

(C) 1 : 2 

(D) 3 : 2 

Answer: (D) 

 

Q. 8 The average of Rajeev’s marks in 7 subjects is 75. His average in six subjects excluding science is 72. How many marks did he get in science ? 

(A) 72 

(B) 90 

(C) 93 

(D) None of these 

Answer: (C) 

 

Q. 9 If 75% a number is added to 75, the result is the number itself. Then the number is 

(A) 400 

(B) 300 

(C) 60 

(D) 50 

Answer: (B) 

 

Q. 10 If a sum of money doubles itself in 8 years at simple interest, the rate percent per annum is 

(A) 11.5 

(B) 12 

(C) 12.5 

(D) 13 

Answer: (C) 

 

Q. 11 If 18 binders bind 900 books in 10 days, how many binders will be required to bind 660 books in 12 days ? 

(A) 22 

(B) 14 

(C) 13 

(D) 11 

Answer: (D) 

 

Q. 12 The largest four digit number which is a perfect cube, is 

(A) 9999 

(B) 9261 

(C) 8000 

(D) None 

Answer: (B) 

Explanation: 

9999 is the largest 4 digit number and 

(20)3 = 8000 

(21)3 = 9261 

This means that the closest cube root of the largest perfect cube is most likely 21. So is the largest perfect cube of four digits. 

=> Ans – (B) 

 

Q. 13 The difference between a two-digit number and the number obtained by interchanging the digits is 27. What is the differences between the digits of the number ? 

(A)

(B)

(C)

(D) Can’t be determined 

Answer: (A) 

 

Q. 14 A mixture of 70 litres of wine and water contains 10% water. How much water must be added to make water 12 and half% of the total mixture ? 

(A) 2 litres 

(B) 10 litres 

(C) 12 litres 

(D) 4 litres 

Answer: (A) 

 

Q. 15 A train of length 150m takes 40.5 seconds to cross a tunnel of length 300m. The speed of the train (in km/hr) is 

(A) 40 

(B) 26⅔  

(C) 13⅓ 

(D) 48 

Answer: (A) 

 

Q. 16 The value of 2342344234 is 

(A) 234

(B) 234

(C) 24

(D) 34

Answer: (A) 

 

Q. 17 The value of (3√2 / √3+√6) – (4√3 / √6+√2) + (√6 / √2+√3) is 

(A) 2

(B)

(C) 3

(D) 6

Answer: (B) 

 

Q. 18 If a2 + b2 + c2 = 2(a-b-c)-3, then the value of 4a – 3b + 5c is 

(A)

(B)

(C)

(D)

Answer: (A) 

 

Q. 19 If 2x + 2/x = 3 then the value of x3 + 1/x2 + 2 is

(A) – 9/8

(B) – 25/8

(C) ⅞ 

(D) 11 

Answer: (C) 

 

Q. 20 Out of the given responses, one of the factors of (a2 − b2)3 + (b2 − c2)3 + (c2 − a2)3 is 

(A) (a + b) (a – b) 

(B) (a + b) (a + b) 

(C) (a – b) (a – b) 

(D) (b – c) (b – c) 

Answer: (A) 

Explanation: 

a2 − b2 b2 − c2 c2 − a2 

Let, X = , Y = , Z = 

a2 − b2 b2 − c2 c2 − a2 

Then, X + Y + Z = 0 (i.e + + = 0) 

We know that, 

3 3 3 

X + Y + Z = 3XYZ i.e, 

(a2 − b2)3 + (b2 − c2)3 + (c2 − a )2 3 a2 − b2)(b2 − c2)(c2 − a2 = 3 ( ) 

One of the factors is, 

a2 − b2(or)(a + b)(a − b) 

Hence, option (A) is the correct answer. 

 

Q. 21 If X = 35 + 2, then the value of x3 – 6x2 + 12x – 13 is 

(A) -1 

(B)

(C)

(D)

Answer: (D) 

 

Q. 22 A tower standing on a horizontal plane subtends a certain angle at a point 160 m apart from the foot of the tower. On advancing 100 m towards it, the tower is found to subtend an angle twice as before. The height of the tower is 

(A) 80 m 

(B) 100 m 

(C) 160 m 

(D) 200 m 

Answer: (A) 

 

Q. 23 ∠A, ∠B, ∠C are three angles of a triangle. If ∠A − ∠B 15∠B − ∠C = 30, = ∠A ∠B, then ∠C, and are 

(A) 80 , 60 , 40 

(B) 70 , 50 , 60 

(C) 80 , 65 , 35 

(D) 80 , 55 , 45 

Answer: (C) 

 

Q. 24 If ABC is an equilateral triangle and D is a point on BC such that AD ⊥ BC, then 

(A) AB : BD = 1 : 1 

(B) AB : BD = 1 : 2 

(C) AB : BD = 2 : 1 

(D) AB : BD = 3 : 2 

Answer: (C) 

 

Q. 25 Sin A + Sin2 A = 1, then the value of cos2 A + cos4 A is 

(A)

(B) 2/3

(C) 1½ 

(D)

Answer: (D) 

Reasoning 

Instructions 

In each of the following questions, select the related word/number from the given alternatives. 

Q. 26 Psychology : Mind : : Arithmetic : ? 

(A) Knowledge 

(B) Number 

(C) Height 

(D) Formulas 

Answer: (B) 

Explanation: 

The study of human mind and its functions is referred as ‘Psychology’ and 

Branch of mathematics dealing with study of numbers is referred as ‘Arithmetics’ Hence, option (B) is the correct answer. 

 

Q. 27 Ice : Coldness : : Earth : ? 

(A) Weight 

(B) Jungle 

(C) Gravitation 

(D) Sea 

Answer: (C) 

Explanation: 

Coldness is the property of Ice whereas Gravity is the property of Earth. 

Hence, option (C) is the correct answer. 

 

Q. 28 Teacher : School : : Nurse : ? 

(A) Doctors 

(B) Patients 

(C) Medicine 

(D) Hospitals 

Answer: (D) 

Explanation: 

Teacher is the person who teaches students in school whereas Nurse is the one who works in hospitals. Hence, option (D) is the correct answer. 

 

Q. 29 11 : 1331 : : 9 : ? 

(A) 979 

(B) 991 

(C) 729 

(D) 879 

Answer: (C) 

Explanation: 

The pattern followed here is, 

113 = 1331 

likewise, 

93 = 729 

Hence, option (C) is the correct answer. 

 

Q. 30 Window : Carpenter :: Statue : ? 

(A) Sculptor 

(B) Mason 

(C) Blacksmith 

(D) Goldsmith 

Answer: (A) 

Explanation: 

Carpenter is the one who makes a window and sculptor is the one who makes a statue. Hence, option (A) is the correct answer. 

Instructions 

In each of the following questions, find the odd number/letters from the given alternatives. 

Q. 31 

(A) XWVU 

(B) SRQP 

(C) NMLK 

(D) EDC(A) 

Answer: (D) 

Explanation: 

Except in option (D) other options are in sequence. 

Hence, option (D) is the correct answer. 

 

Q. 32 

(A) 24 

(B) 56 

(C) 84 

(D) 94 

Answer: (D) 

Explanation: 

Every number except 94 is divisible by ‘4’ 

Hence, option (D) is the correct answer. 

Instructions 

For the following questions answer them individually 

 

Q. 33 Which of the given responses would be a meaningful order of the following in ascending order ? 

(A) 0640 hrs 

(B) 1930 hrs 

(c)  1335 hrs 

(D) 2000 hrs 

Answer:

 

Q. 34 Which one set of letters when sequentially placed at the gaps in the given letter series shall complete it ? 

B_f_ _ _ ndfg _ 

(A) dgggb 

(B) dgbg 

(C) bgdgg 

(D) gdggb 

Answer: (D) 

Instructions 

In each of the following questions, a series is given, with one term missing. Choose the correct alternative from the given ones that will complete the series. 

 

Q. 35 

(A)

(B) -12 

(C) 12 

(D)

Answer: (B) 

Explanation: 

The pattern followed here is, 

2 x 3 x 5 = 30, 

5 x 1 x -1 = -5, 

4 x 3 x -1 = -12 

(i.e all the 3 numbers are multiplied) 

Hence, option (B) is the correct answer. 

 

Q. 36 50, 65, 82, ?, 122 

(A) 101 

(B) 97 

(C) 105 

(D) 100 

Answer: (A) 

Explanation: 

The pattern followed here is, 

50 + 15 = 65, 

65 + 17 = 82, 

82 + 19 = 101, 

101 + 21 = 122. 

Hence, option (A) is the correct answer. 

Instructions 

For the following questions answer them individually 

 

Q. 37 In a group of 20 people, 8 read Hindi, 11 read English while 5 of them read none of these two. How many of them read Hindi and English both ? 

(A)

(B)

(C)

(D)

Answer: (C) 

 

Q. 38 Unscramble the following letters to frame a meaningful word and find out the correct numerical sequence of the letters. 

R E S T A U R A N T 

1 2 3 4 5 6 7 8 9 10 

(A) 10 2 3 5 16 4 7 8 9 

(B) 3 1 2 4 5 7 6 9 8 10 

(C) 1 3 5 2 9 4 8 6 7 10 

(D) 9 1 3 6 2 7 5 4 8 10 

Answer: (B) 

 

Q. 39 If ‘DICTIONARY’ is coded as 5479482361, then ‘YARD’ can be coded as ? 

(A) 1653 

(B) 1635 

(C) 1536 

(D) 1365 

Answer: (D) 

Explanation: 

In the given code language, alphabets are directly related to the numbers on the right. 

D = 5 ; I = 4 ; C= 7 ; T = 9 ; O = 8 ; N = 2 ; A = 3 ; R = 6 ; Y = 1. 

Hence, code for YAR(D) is ‘1365’ 

Hence, option (D) is the correct answer. 

 

Q. 40 In a row of students Ganesh is 7th from one extreme and 11th from the other. Find the total numbers of students in the row. 

(A) 17 

(B) 18 

(C) 19 

(D) 20 

Answer: (A) 

Explanation: 

As Ganesh is 7th from one extreme end (say left end), number of students to his left will be ‘6’ and also, He is 11th from other end (say right end), number of students to his right will be ’10’. 

Total number of students = 6 + 1 + 10 = 17. 

Hence, option (A) is the correct answer. 

 

Q. 41 Find the correct answer for the unsolved equation 

5 x 6 x 3 = 356, 1 x 0 x 5 = 510, 5 x 6 x 7 = ? 

(A) 567 

(B) 657 

(C) 210 

(D) 756 

Answer: (D) 

Explanation: 

Last digit is kept at first, first digit is kept at second, second digit is kept at last. 

Code for 5 x 6 x 7 will be ‘756’ 

Hence, option (D) is the correct answer. 

 

Q. 42 A, B, C and D are playing a game of carrom. A, C and B, D are partners. C is to the left of D who is facing South. Then A is facing 

(A) North 

(B) South 

(C) East 

(D) West 

Answer: (C) 

Explanation: 

As per the given question, final arrangement will be 

A is facing towards East in the above arrangement. 

Hence, option (C) is the correct answer. 

 

Q. 43 Select the answer figure in which the question figures are hidden / embedded. 

Answer: (A) 

 

Q. 44 Which of the following diagram best depicts the relationship between student, college and school ? 

Answer: (B) 

Explanation: 

A student can be in a school or in a college but not in both at the same time. 

Hence, option (B) is the correct answer. 

 

Q. 45 Statement is given followed by three conclusions I, II and III. You have to consider the statement to be true even they seem to be at variance from commonly known facts. You have to decide which of the given conclusions, if any, follows from the given statement. Statement: Comic books contain pictures 

Conclusions: 

I. All books contain pictures 

II. Books may or may not contain pictures 

III. Books other than the comic books does not contain pictures. 

(A) Only Conclusion I follows 

(B) Only Conclusion II follows 

(C) Both Conclusions I and II follows 

(D) Neither Conclusion follows 

Answer: (B) 

 

Q. 46 Which one of the answer figure shall complete the given question figure ? 

Answer: (B) 

Explanation: 

The first diagram has only one dot. Second diagram which is below it, has two dots.Third diagram has three dots. Following the same pattern, fourth diagram must contain four dots in it. 

Hence, option (B) is the correct answer. 

 

Q. 47 From the given answer figures, select the one in which the question figure is hidden / embedded 

Answer: (B) 

 

Q. 48 A piece of paper is folded and cut as shown below in the question figures. From the given answer figures, indicate how it will appear when opened. 

Answer: (B) 

 

Q. 49 Which of the answer figures is the right images of the given figure ? 

P R A Y E R 

Answer: (C) 

Explanation: 

Word : P R A Y E R 

In the image of the word, the letters will swap position, i.e. first letter will come at end, second at second last and so on, thus first and last options are not possible. Also, direction of the letters will also be reversed. 

=> Ans – (C) 

 

Q. 50 A word is represented by only one set of numbers as given in any one of the alternatives. The set of numbers given in the alternatives are represented by two classes of alphabets as in two matrices given below. The column and rows of Matrix – I are numbered from 0 to 4 and that of Matrix – II are numbered from 5 to 9. A letter from these matrices can be represented first by its row and next by its column, e.g., A’ can be represented by 01, 13 etc., and S’ can be represented by 55, 67 etc. Similarly, you have to identify the set for the letters given. 

(A) 65, 23, 14, 55 

(B) 86, 34, 42, 69 

(C) 78, 41, 23, 86 

(D) 57, 11, 33, 96 

Answer: (D) 

General Awareness 

Instructions 

For the following questions answer them individually 

Q. 51 Earthquakes are caused by ? 

(A) Tectonism 

(B) Denudation 

(C) Earth’s revolution 

(D) Earth’s rotation 

Answer: (A) 

 

Q. 52 When the days and nights are equal, the rays of the sum directly fall on the ? 

(A) Equator 

(B) Tropic of Cancer 

(C) South Pole 

(D) North Pole 

Answer: (B) 

 

Q. 53 Which of the following rivers crosses the equator twice ? 

(A) Amazon 

(B) Nile 

(C) Congo 

(D) Orinoco 

Answer: (C) 

 

Q. 54 Who discovered the sea route to India? 

(A) Vascoda Gama in 1498 

(B) Columbus in 1402 

(C) Magellan in 1506 

(D) Sir Hopkins in 1698 

Answer: (A) 

 

Q. 55 Who wrote As you like ? 

(A) Bernard Shaw 

(B) Shankespear 

(C) Leotolstoy 

(D) Mulk Raj Anand 

Answer: (B) 

 

Q. 56 Who were the three statesmen who formulated NAM ? 

(A) Tito, Nasser and Bhutto 

(B) Nehru, Nasser and Tito 

(C) Nasser, Tito and Nehru 

(D) Nehru, Chouen-Lai and Bhutto 

Answer: (A) 

 

Q. 57 UNESCO stands for ? 

(A) United Nations Ecological Science Co-operation 

(B) Union of National Educational Scientific and Cultural Organisation 

(C) United Nations Educational Scientific and Cultural Organisation 

(D) None of these 

Answer: (C) 

 

Q. 58 The greatest painter of birds at Jahangir’s court was ? 

(A) Khwaja Abdus Samad 

(B) Syed Ali Tabrizi 

(C) Basawan 

(D) Mansur 

Answer: (D) 

 

Q. 59 Huen-tsang found Jainsim flourishing in ? 

(A) Orissa 

(B) Kashmir 

(C) Bengal 

(D) Bihar 

Answer: (C) 

 

Q. 60 ‘Swaraj is my birth right and I shall have it”. This was advocated by ? 

(A) Lala Lajpat Rai 

(B) Lokmanya Tilak 

(C) Mahatma Gandhi 

(D) Sardar Patel 

Answer: (B) 

 

Q. 61 1st partition of Bengal under Lord Curzon took place in ? 

(A) 1901 

(B) 1915 

(C) 1905 

(D) 1907 

Answer: (C) 

 

Q. 62 Who proposed the chemical evolution of life ? 

(A) Darwin 

(B) Lammarck 

(C) Oparin 

(D) Haechel 

Answer: (C) 

 

Q. 63 Water has Maximum density at ? 

(A) Latex1 

(B) Latex2 

(C) Latex3 

(D) Latex4 

Answer: (B) 

 

Q. 64 Which of the following chemicals is used in photograph ? 

(A) Aluminium hydroxide 

(B) Silver bromide 

(C) Potassium nitrate 

(D) Sodium chloride 

Answer: (B) 

 

Q. 65 Human blood contains – percentage of plasma ? 

(A) 35% 

(B) 40% 

(C) 50% 

(D) 55% 

Answer: (D) 

 

Q. 66 Rickets is a disease of the ? 

(A) bones 

(B) tissue 

(C) muscles 

(D) blood 

Answer: (A) 

 

Q. 67 Trachoma is disease of the ? 

(A) Liver 

(B) Eyes 

(C) Lungs 

(D) Kidneys 

Answer: (B) 

 

Q. 68 Who is regarded as the architect of the Indian Constitution ? 

(A) B.N. Rao 

(B) Dr B.R. Ambedkar 

(C) N.G. Ayyangar 

(D) Dr Rajendra Prasad 

Answer: (B) 

 

Q. 69 What is the salary of the Chief Justice of India ? 

(A) ₹1,60,000 

(B) ₹80,000 

(C) ₹1,00,000 

(D) ₹75,000 

Answer: (B) 

 

Q. 70 How many states in India have two Houses, Viz., Legislative Assembly and Legislative Council ? 

(A)

(B)

(C)

(D) 10 

Answer: (A) 

 

Q. 71 The works ‘Socialist’ and Secular’ were added to the Preamble of the Constitution of India by the__Constitution Amendment Act 1976 ? 

(A) 44th 

(B) 42nd 

(C) 32nd 

(D) 9th 

Answer: (B) 

 

Q. 72 The typical area of ‘sal’ forest in the Indian Peninsular upland occurs ? 

(A) on the Western Ghats 

(B) between the Tapti and the Narmada 

(C) to the north-east of the Godavari 

(D) on the Malwa Plateau 

Answer: (A) 

 

Q. 73 Which coloured ring represents the Asian Continent in the Olympics Emblem ? 

(A) Blue 

(B) Yello 

(C) Red 

(D) Green 

Answer: (B) 

 

Q. 74 The first Indian to share the Oscar Award was ? 

(A) Hargobind Khorana 

(B) Rabindranath Tagore 

(C) Bhanu Athaiya 

(D) None of these 

Answer: (C) 

 

Q. 75 The headquarters of Asian Development Bank is located in which of the following cities ? 

(A) Jakarta 

(B) Singapore 

(C) Bangkok 

(D) Manila 

Answer: (D) 

English 

Instructions 

Some parts of the sentences have errors and some have none. Find out which part of a sentence has an error, the appropriate letter (1, 2, 3). If a sentence is free error, (4) is the Answer Sheet. 

Q. 76 

(A) Either he 

(B) or his wife 

(C) are coming to attend the dinner 

(D) No error 

Answer: (C) 

Q. 77 

(A) The launch of the first artificial satellite by the Russians 

(B) took the world almost entirely unawares 

(C) and provocation flood of speculations about its significances 

(D) No error 

Answer: (B) 

Instructions 

Sentence are given with blanks to be filled in with an appropriate word(s). Four alternatives are suggested for each question. Choose the correct alternative out of the four and indicate itin the Answer Sheet. 

Q. 78 ………….the rain forests is very important, if we do not want the flora and fauna found there to become extinct. 

(A) Reserving 

(B) Destroying 

(C) Preserving 

(D) Maintaining 

Answer: (C) 

 

Q. 79 If I, had helped him, he………… 

(A) will not be drowned 

(B) would not be drowned 

(C) will not have drowned 

(D) would not have drowned 

Answer: (D) 

 

Q. 80 When will you hand………….your assignment ? 

(A) in 

(B) back 

(C) down 

(D) into 

Answer: (A) 

Instructions 

Out of the alternatives, choose the one which best expresses the meaning of the given word and mark it in the Answer Sheet. 

Q. 81 INTREPID

(A) hesitant 

(B) fearless 

(C) extrovert 

(D) rash 

Answer: (B) 

 

Q. 82 PRODIGAL 

(A) exclusive 

(B) productive 

(C) lavish 

(D) carefree 

Answer: (C) 

 

Q. 83 PERSPICUOUS 

(A) relevant 

(B) precise 

(C) brief 

(D) clear 

Answer: (D) 

Instructions 

In questions choose the word opposite in meaning to the given word and mark it in the Answer Sheet. 

 

Q. 84 ELEVATION 

(A) reduction 

(B) humiliation 

(C) depression 

(D) debasement 

Answer: (C) 

 

Q. 85 GLOSSY 

(A) dull 

(B) shining 

(C) weary 

(D) tired 

Answer: (A) 

 

Q. 86 APPROPRIATE 

(A) dissimilar 

(B) incomparable 

(C) unsuitable 

(D) disparate 

Answer: (C) 

Instructions 

Four alternatives are given for the idiom/phrase underlined in the sentences. Choose the alternative which best expresses the meaning of the idiom/phrase and mark it in the Answer Sheet. 

Q. 87 To take someone for a ride 

(A) to give a ride to someone 

(B) to deceive someone 

(C) to be indifferent 

(D) to disclose a secret 

Answer: (B) 

 

Q. 88 To move heaven and earth 

(A) to cause an earthquake 

(B) to try everything possible 

(C) to pray to all gods 

(D) to travel in a rocket 

Answer: (B) 

 

Q. 89 To smell a rat 

(A) to smell foul 

(B) to see a rat 

(C) to chase a rat 

(D) to be suspicious 

Answer: (D) 

Instructions 

A part of the sentence is underlined. Below are given alternatives to the underlined part at 1, 2, and 3 which may improve the sentence. Choose the correct alternatives. In case no improvement is needed, your answer is 4. 

 

Q. 90 The courtiers used to tell the King how efficient an administrator he was all day long. 

(A) The courtiers all day long used to tell the King how, efficient an administrator he was 

(B) The courtiers and used all day long to tell the King how efficient an administrator he was 

(C) The courtiers used to tell the King all day long how efficient an administrator he was 

(D) No improvement 

Answer: (C) 

 

Q. 91 Every Saturday I go out for shopping 

(A) for shops 

(B) to shopping 

(C) for shop 

(D) No improvement 

Answer: (D) 

 

Q. 92 We had a grand party and we enjoyed very much. 

(A) We had a grand party and enjoyed very much 

(B) We had a grand party to enjoy very much 

(C) We had a grand party and we enjoyed ourselves 

(D) No improvement 

Answer: (C) 

Instructions 

Out of the four alternatives, choose the one which can be substituted for the given words/sentence. 

Q. 93 Release of prisoner from jail on certain terms and conditions. 

(A) Parole 

(B) Parley 

(C) Pardon 

(D) Acquittal 

Answer: (A) 

 

Q. 94 Loss of memory 

(A) Ambrosia 

(B) Amnesia 

(C) Insomnia 

(D) Forgetting 

Answer: (B) 

 

Q. 95 To struggle helplessly 

(A) Flounder 

(B) Founder 

(C) Fumble 

(D) Finger 

Answer: (A) 

Instructions 

The animal mind is like a telephone exchange, it receives stimuli from outside through the sense organs and sends out appropriate responses through the nerves that govern muscles, glands and other parts of the body. The organism is constantly interacting with its surroundings receiving messages and acting on the new state of affairs that the messages signify. 

But the human mind is not a simple transmitter like a telephone exchange. It is more like a great projector; for instead of merely mediating between an event in the outer world and a creature’s responsive action, it transforms or, if you will, distorts the event into an image to be looked at, retained an contemplated. For the images of things we remember are not exact and faithful transcriptions even of our actual sense impressions. The are made as much by what we think as by what we see. It is a well-known fact that if you ask several people the size of moon’s disk as they look at it, their estimates will very from the area of dime to that of a barrel top. Like a magic lantern, the mind projects its ideas of things on the screen of what we call ‘memory’ : but like all projections, these ideas are transformations of actual things. They are in fact, symbols of reality, not pieces of it. 

Q. 96 An animal mind and a human mind differ like 

(A) a telephone exchange and a projector 

(B) a screen and an image 

(C) a lantern and a candle 

(D) projections and illusions 

Answer: (A) 

 

Q. 97 Human memory is subject to 

(A) thought and visual impression 

(B) thought and reasoning 

(C) fancy and fantasy 

(D) nothing but hard training 

Answer: (A) 

Instructions 

In each of the following Questions, a sentence has been given in Active/passive voice out of the four Alternatives suggested, select the one which best expresses the same sentence in Passive/Active voice. 

Q. 98 A good deal of money will be made by the investment. 

(A) That investment has made a good deal of money 

(B) That investment will be making a good deal of money 

(C) That investment had been making a good deal of money 

(D) That investment has been making a good deal of money 

Answer: (D) 

 

Q. 99 Who will help me ? 

(A) By whom I shell be helped ? 

(B) By whom will be I helped ? 

(C) By whom would I be helped ? 

(D) By whom I will be helped ? 

Answer: (B) 

 

Q. 100 In the following Questions, four words are given, out of which only one words is correctly spelt find the correctly spelt word. 

(A) vaccum 

(B) vacuum 

(C) vacum 

(D) vaccuum 

Answer: (B) 

SSC GD 2012 Shift-I Previous Year Paper

SSC GD 2012 Shift I 

Reasoning 

Instructions 

In each of the following questions, select the related word/letters /number from the given alternatives. 

Q. 1 Eye: Cataract : : Skin : ? 

(A) Pyorrhea 

(B) Sinusitis 

(C) Eczema 

(D) Trachoma 

Answer: (C) 

Explanation: 

Cataract is a clouding of the lens in the eye which leads to a decrease in vision, similarly, eczema is a medical condition in which patches of skin become rough and inflamed with blisters which cause itching and bleeding. 

=> Ans – (C) 

 

Q. 2 Vitamin A : Carrot :: Vitamin C : ? 

(A) Meat 

(B) Fish 

(C) Egg 

(D) Orange 

Answer: (D) 

Explanation: 

Carrot is a good source of vitamin A whereas Orange is a good source of vitamin C. 

Hence, option (D) is the correct answer. 

 

Q. 3 ACF : GIL :: MOR : ? 

(A) SUX 

(B) TUX 

(C) UWZ 

(D) SVY 

Answer: (A) 

Explanation: 

The pattern followed here is, 

A + 6 = G, C + 6 = I, F + 6 = L 

Code for MOR will be, 

M + 6 = S, O + 6 = U, R + 6 = X 

Hence, option (A) is the correct answer. 

 

Q. 4 32 : 66: : 134 : ? 

(A) 271 

(B) 268 

(C) 270 

(D) 275 

Answer: (C) 

Explanation: 

The pattern followed here is, 

(32 x 2) + 2 = 66. 

Therefore, code for 134 will be, 

(134 x 2) + 2 = 270. 

Hence, option (C) is the correct answer. 

Instructions 

In the following questions, find the odd word/number from the given alternatives. 

Q. 5 

(A) Sweater 

(B) Muffler 

(C) Socks 

(D) Shawl 

Answer: (C) 

Explanation: 

A sock is a type of clothing worn on feet and all other clothing’s mentioned in the options are worn on upper parts of the body. Hence, option (C) is the correct answer. 

 

Q. 6 

(A) QWBS 

(B) MPTD 

(C) UIAE 

(D) RVGW 

Answer: (C) 

Explanation: 

Except in option C, other options contain consonants in them. (Only Option C) contains vowels in it) 

Hence, option (C) is the correct answer. 

 

Q. 7 

(A) 1942 

(B) 1937 

(C) 1935 

(D) 1925 

Answer: (A) 

Explanation: 

Except ‘1942’ other numbers are odd numbers. 

Hence, option (A) is the correct answer. 

Instructions 

For the following questions answer them individually 

 

Q. 8 Arrange the leaves according to their size (Small to large): 

1. Mango leaf 

2: Tamarind leaf 

3: Papaya leaf 

4. Banana leaf 

(A) 1, 2, 3, 4 

(B) 3, 2, 4, 1 

(C) 2, 1, 3, 4 

(D) 2, 3, 1, 4 

Answer: (C) 

Explanation: 

The size of the leaf’s in ascending order would be, 

1) Tamarind 2) Mango 3) Papaya 4) Banana 

Hence, option (C) is the correct answer. 

 

Q. 9 In the following questions, a series is given with one term missing. Choose the correct alternative form the given ones that will complete the series. 

N, P, R, ? 

(A)

(B)

(C)

(D)

Answer: (A) 

Explanation: 

The pattern followed here is, 

N + 2 = P 

P + 2 = R 

R + 2 = T 

Hence, option (A) is the correct answer. 

 

Q. 10 In the following questions, a series is given with one term missing. Choose the correct alternative form the given ones that will complete the series. 

1 4 9 16 

25 36 49 ? 

81 100 ? 144 

(A) 64 & 121 

(B) 20 & 100 

(C) 121 & 46 

(D) 95 & 150 

Answer: (A) 

Explanation: 

The given numbers are the squares of natural numbers starting from ‘1’ 

Hence, missing numbers are squares of 8 and 11 i.e 64 and 121. 

Hence, option (A) is the correct answer. 

 

Q. 11 In the following questions, a series is given with one term missing. Choose the correct alternative form the given ones that will complete the series. 

4, 3, 2.5, 2.25, ? 

(A)

(B) 1.125 

(C)

(D) 2.125 

Answer: (D) 

Explanation: 

The pattern followed here is, 

4 – 1 = 3, 

3- 0.5 = 2.5, 

2.5 – 0.25 = 2.25, 

2.25 – 0.125 = 2.125. 

Hence, option (D) is the correct answer. 

 

Q. 12 If 8th of April falls on Monday, what would be the 30th day of that month ? 

(A) Sunday 

(B) Monday 

(C) Tuesday 

(D) Wednesday 

Answer: (C) 

Explanation: 

If 8th of April falls on Monday then 15th, 22nd, 29th will also be Monday. Therefore, 30th of that month will be Tuesday. Hence, option (C) is the correct answer. 

 

Q. 13 From the given alternatives. Select the word which cannot be formed using the letters of the given word. 

ESTABLISHMENT 

(A) TABLE 

(B) BLUNT 

(C) TENTS 

(D) STATE 

Answer: (B) 

Explanation: 

Except the word ‘BLUNT’ other words given in the options can be formed using “ESTABLISHMENT” There is no ‘U’ in the given word “ESTABLISHMENT”. 

Hence, option (B) is the correct answer. 

 

Q. 14 If SUNDAY is coded as 012345 and BIG is coded as 678, how would you encode SANDBAY ? 

(A) 0234456 

(B) 0423645 

(C) 0432645 

(D) 0342456 

Answer: (B) 

Explanation: 

SUNDAY is coded as ‘012345’ and BIG is coded as ‘678’ 

Each alphabet on the left is directly related to each number on the right. 

S = 0 ; U = 1 ; N = 2 ; D = 3 ; A = 4 ; Y = 5 ; B = 6 ; I = 7 ; G = 8 

In the same way, 

SANDBAY is coded as ‘0423645’ 

Hence, option (B) is the correct answer. 

 

Q. 15 Select the correct combination of mathematical signs to replace 

* signs and to balance the following equation 

9 * 3 * 3 * 3 * 6 

(A) ÷ x – = 

(B) + – x = 

(C) – + + = 

(D) x + – = 

Answer: (A) 

Explanation: 

The correct combination would be, 

9 ÷ 3 x 3 – 3 = 6 

Hence, option (A) is the correct answer. 

 

Q. 16 Select the correct response. If 

RAJ = 29, EDUCATION = ? 

(A) 85 

(B) 86 

(C) 88 

(D) 92 

Answer: (D) 

Explanation: 

The pattern followed here is, 

R(18) + A(1) + J(10) = 29 

The code for EDUCATION will be, 

E(5) + D(4) + U(21) + C(3) + A(1) + T(20) + I(9) + O(15) + N(14) = 92 

Hence, option (D) is the correct answer. 

 

Q. 17 At dusk, Rohit started walking facing the west, After a while, he met his friend and both turned to their left. They halted for a while and started moving by turning again to their right. Finally Rohit waved ‘good bye’ to his friend and took a left turn at a corner. At which direction is Rohit moving now ? 

(A) South 

(B) West 

(C) North 

(D) East 

Answer: (A) 

Explanation: 

As per the given question, 

Finally, Rohit is facing South. 

Hence, option (A) is the correct answer. 

 

Q. 18 Find the number of triangles in the given figure 

(A)

(B)

(C)

(D)

Answer: (B) 

 

Q. 19 Which figure best represents the relation among Man, Vegetables and Cow ? 

Answer: (A) 

Explanation: 

All three given entities are completely different from each other. 

Hence, option (A) is the correct answer. 

 

Q. 20 There are two statements labelled as Assertion A and Reason R.

A. A little gap is left between iron rails. 

R. Iron expands is summer. 

(A) Both A and R are true. 

(B) Both A and R are false. 

(C) A is true and R is false. 

(D) A is false and R is true. 

Answer: (A) 

 

Q. 21 Which answer figure will complete the pattern in the question figure: 

Answer: (B) 

 

Q. 22 From the given answer figures, select the one in which the question figure is hidden / embedded. 

Answer: (A) 

 

Q. 23 A piece of paper is folded and cut as shown below in the question figures. From the given answer figures, indicate how it will appear when opened. 

Answer: (B) 

 

Q. 24 If a mirror is placed on the line MN, then which of the answer figures is the right image of the given figure ? 

Answer: (B) 

 

Q. 25 A word is represented by only one set of numbers as given in any one of the alternatives. The sets of numbers given in the alternatives are represented by two classes of alphabets as in two matrices given below. The columns and rows of Matrix-I are numbered from 0 to 4 and that of Matrix-II are numbered form 5 to 9. A letter from these matrices can be represented first by its row and next by its column, e.g. ‘A’ can be represented by00, 23, etc. and ‘P’ can be represented by 55, 69, etc. Similarly you have to identify the set for the word given in the question. 

BEAST 

(A) 33, 42, 58, 55, 87 

(B) 31, 68, 32, 55, 95 

(C) 24, 22, 23, 58, 59 

(D) 42, 31, 10, 13, 77 

Answer: (C) 

Explanation: 

As per the given question, 

(B) can be represented by – 01, 10, 24, 33, 42. 

Similarly, 

E – 04, 13, 22, 31, 40 

(A) – 00, 14, 23, 32, 41. 

S – 58, 67, 76, 85, 99. 

T – 59, 68, 77, 86, 95. 

24, 22, 23, 58, 59 is one of the combinations for BEAST. 

Hence, option (C) is the correct answer. 

General Awareness 

Instructions 

For the following questions answer them individually 

Q. 26 Production function refers to the functional relationship between input and______. 

(A) product 

(B) produce 

(C) output 

(D) service 

Answer: (C) 

 

Q. 27 ‘Self Reliance’ was the main objective of 

(A) Fourth Plan 

(B) Seventh Plan 

(C) Third Plan 

(D) Sixth Plan 

Answer: (D) 

 

Q. 28 District Judge is under the control of 

(A) State Government 

(B) High Court 

(C) Supreme Court 

(D) Governor 

Answer: (B) 

 

Q. 29 What is the meant by social justice ? 

(A) All should have same economic rights 

(B) All should have same political rights 

(C) All kinds of discrimination based on caste, creed, colour and sex should be eliminated 

(D) All should be granted right to freedom of religion 

Answer: (C) 

 

Q. 30 _______ are essential for liberty. 

(A) Restrictions 

(B) Rights 

(C) Privileges 

(D) Laws 

Answer: (B) 

 

Q. 31 Name the country which launched the first Satellite “Sputnik” into the space. 

(A) United States of America 

(B) Soviet Union 

(C) Japan 

(D) England 

Answer: (B) 

 

Q. 32 Who is commonly known as the Iron Man ? 

(A) Sardar Vallabh Bhai Patel 

(B) Vittal Bhai Patel 

(C) Bal Gangadhar Tilak 

(D) Bipin Chandra Pal 

Answer: (A) 

 

Q. 33 Mahavira’s first disciple was 

(A) Bhadrabahu 

(B) Sthulabhadra 

(C) Charvaka 

(D) Jamali 

Answer: (D) 

 

Q. 34 Which of the following is the most numerous tribe in India ? 

(A) Todas 

(B) Bhils 

(C) Garos 

(D) Gonds 

Answer: (D) 

 

Q. 35 Soils of Western Rajasthan have a high content of 

(A) Aluminium 

(B) Calcium 

(C) Nitrogen 

(D) Phosphrous 

Answer: (B) 

 

Q. 36 Activity of an enzyme can be modulated by change of ? 

(A) pH 

(B) Light 

(C) Humidity 

(D) Rainfall 

Answer: (A) 

 

Q. 37 Proteins are digested by 

(A) Proteases 

(B) Amylases 

(C) Lipases 

(D) Nucleases 

Answer: (A) 

 

Q. 38 Jaundice is a disease which effects 

(A) Heart 

(B) Liven 

(C) Spleen 

(D) Gall bladder 

Answer: (B) 

 

Q. 39 Which metal is the heaviest in periodic table among the following ? 

(A) Os 

(B) Pt 

(C) Pb 

(D)

Answer: (A) 

 

Q. 40 The chemical formula of the laughing gas is 

(A) NO 

(B) N2

(C) NO2 

(D) N2O3 

Answer: (B) 

 

Q. 41 Outside of cooking utensils are generally left black from below because 

(A) it is difficult to clean daily 

(B) black surface is a good conductor of heat 

(C) black suface is a poor conductor of heat 

(D) black surface is a good absorber of heat 

Answer: (D) 

 

Q. 42 The colour of sky appears blue due to 

(A) reflection 

(B) refraction 

(C) scattering of shorter wave lengths 

(D) dispersion 

Answer: (C) 

 

Q. 43 GUI stands for 

(A) Graphical User Interface 

(B) Graphical User Information 

(C) Graphical User Interaction 

(D) Graphical User Instruction 

Answer: (A) 

 

Q. 44 Indian Council of Forestry Research and Education is located in 

(A) Dehradum 

(B) Ranchi 

(C) New Delhi 

(D) Raipur 

Answer: (A) 

 

Q. 45 Cholesterol is absent in 

(A) Groundnut oil 

(B) Butter oil 

(C) Butter milk 

(D) Ice cream 

Answer: (A) 

 

Q. 46 India’s first Crypto-Currency exchange “coincome” has launched by which payment gateway ? 

(A) Direcpay 

(B) Citrus pay 

(C) Payzippy 

(D) Billdesk 

Answer: (D) 

 

Q. 47 “Dandia” is a popular dance of 

(A) Gujarat 

(B) Assam 

(C) Jharkhand 

(D) Maharashtra 

Answer: (A) 

 

Q. 48 ‘Natya Shastra was written by 

(A) Bharat Muni 

(B) Narad Muni 

(C) Jandu Muni 

(D) Vyas Muni 

Answer: (A) 

 

Q. 49 Vardhaman Mahavir is also known as 

(A) Jena 

(B) Great teacher 

(C) Great preacher 

(D) Jain 

Answer: (A) 

 

Q. 50 “Ranji Trophy” is associated with 

(A) Hockey 

(B) Football 

(C) Cricket 

(D) Kabaddi 

Answer: (C) 

Quant 

Instructions 

For the following questions answer them individually 

Q. 51 The value of 0.65 × 0.65 + 0.35 × 0.35+0.70 × 0.65 is 

(A) 1.75 

(B) 1.00 

(C) 1.65 

(D) 1.55 

Answer: (B) 

 

Q. 52 How many numbers between 400 and 800 are divisible by 4, 5 and 6 ? 

(A)

(B)

(C)

(D) 10 

Answer: (A) 

 

Q. 53 If sum of two number be a and their product be, b, then the sum of their reciprocals is 

(A) 1/a + 1/b

(B) b/a

(C) a/b 

(D) 1/ab

Answer: (C) 

 

Q. 54 In a camp of 160 students provisions are available for 10 days. If 40 more students join the camp, how long will the provisions last ? 

(A)

(B) 6½ 

(C)

(D) 12½ 

Answer: (C) 

 

Q. 55 Three taps A, B, C can fill an overhead tank in 4, 6 and 12 hours respectively. How long would the three taps take to fill the tank if all of them are opened together ? 

(A) 2 hrs. 

(B) 4 hrs 

(C) 3 hrs 

(D) 5 hrs 

Answer: (A) 

 

Q. 56 The perimeter of a triangle and an equilateral triangle are same. Also, one of the sides of the rectangle is equal to the side of the triangle. The ratio of the areas of the rectangle and the triangle is 

(A) 1 : √3

(B) 1 : √3

(C) 2 : √3

(D) 4 : √3

Answer: (C) 

 

Q. 57 A solid spherical copper ball, whose diameter is 14 cm, is melted and converted into a wire having diameter equal to 14 cm. The length of the wire is 

(A) 27 cm 

(B) 16/3 

(C) 15 cm 

(D) 28/3 

Answer: (D) 

 

Q. 58 Discount on a pair of shoes marked at Rs.475 and discounted at 15% is 

(A) Rs. 70 

(B) Rs.71.25 

(C) Rs.72 

(D) Rs.72.25 

Answer: (B) 

 

Q. 59 The cost price of an article is Rs.100. (A) discount series of 5%, 10% successively reduces the price of a article by 

(A) Rs 4.5 

(B) Rs 14.5 

(C) Rs 24.5 

(D) None of the above 

Answer: (B) 

 

Q. 60 A grinder was marked at Rs.3,600/ After given a discount of 10% the dealer made a profit of 8%. Calculate the cost price. 

(A) Rs. 3,000 

(B) Rs.3,312 

(C) Rs.3,240 

(D) Rs.2,960 

Answer: (A) 

 

Q. 61 If = 6xy, then x: y is x2 + 9y2 

(A) 1 : 3 

(B) 3 : 2 

(C) 3 : 1 

(D) 2 : 3 

Answer: (C) 

 

Q. 62 In a school 1/10 of the boys are same in number as 1/4 of the girls and 5/8 of the girls are same in number as ¼ of the boys. The ratio of the boys to girls in that school is 

(A) 2 : 1 

(B) 5 : 2 

(C) 4 : 3 

(D) 3 : 2 

Answer: (B) 

 

Q. 63 The average of 8 numbers is 27. If each of the numbers is multiplied by 8, find the average of new set of numbers. 

(A) 1128 

(B) 938 

(C) 316 

(D) 216 

Answer: (D) 

 

Q. 64 In a prep school, the average weight of 30 girls in a class among 50 students is 16 kg and that of the remaining students is 15.5 kg. What is the average weight of all the students in the class ? 

(A) 15.2 kg 

(B) 15.8 kg 

(C) 15.4 kg 

(D) 15.6 kg 

Answer: (B) 

 

Q. 65 The average age of a husband and his wife was 23 years at the beginning of their marriage. After five years they have a one-year old child. The average age of the family of three, when the child was born, was 

(A) 27 years 

(B) 24 years 

(C) 18 years 

(D) 20 years 

Answer: (C) 

 

Q. 66 If the profit on sale price be 20%, the percentage of profit on cost price is 

(A) 20% 

(B) 30% 

(C) 22% 

(D) 25% 

Answer: (D) 

 

Q. 67 A shopkeeper purchased a TV for Rs.2,000 and a radio for Rs.750. He sells the TV at a profit of 20% and ther radio at a loss of 5%. The total loss or gain is 

(A) Gain Rs.353.50 

(B) Gain Rs.362.50 

(C) Loss Rs.332 

(D) Loss Rs.300 

Answer: (B) 

 

Q. 68 A container containing 400 litres of oil lost 8% by leakage. Oil left in the container is 

(A) 320 litres 

(B) 368 litres 

(C) 332 litres 

(D) 32 litres 

Answer: (B) 

 

Q. 69 If x2– 3x + 1 = 0, then the value of  x5 + 1/x5 is equal to 

(A) 87 

(B) 123 

(C) 135 

(D) 201 

Answer: (B) 

 

Q. 70 The value of cosec2 is 18− 1/cot 72 2  

(A) 1/√3

(B) √2/3

(C) 1/2

(D) 1

Answer: (D) 

 

Q. 71 The elevation of the top of a tower from a point on the ground is 45. On travelling 60 m from the point towards the tower, the alevation of the top becomes 60 . The height of the tower, in metres, is 

(A) 30 

(B) 30(3- √3) 

(C) 30(3 + √3) 

(D) 30 3 

Answer: (C) 

 

Q. 72 (a-b)2 / (b-c) (c-a) + (b-c)2 / (a-b) (c-a) +(a-c)2 / (a-b) (b-c) , a ≠ b c, is 

(A)

(B)

(C)

(D)

Answer: (D) 

 

Q. 73 The product of two numbers is 2160 and their HCF is 12. Numbers of such possible pairs is 

(A)

(B)

(C)

(D)

Answer: (B) 

Instructions 

The following table gives the result of a survey based on newspaper reading habits. Study the table and answer the questions. 

Q. 74 The number of people who read only English newspapers. 

(A) 975 

(B) 654 

(C) 1086 

(D) 221 

Answer: (B) 

Explanation: 

Number of people who read only English newspapers 

= 123 + 206 + 325 = 654 

=> Ans – (B) 

 

Q. 75 The total number of people surveyed are 

(A) 2040 

(B) 1086 

(C) 12961 

(D) 1936 

Answer: (D) 

Explanation: 

Number of people who do not read newspapers = 162 + 13 + 21 = 196 

Number of people who read only in regional languages = 271 + 285 + 209 = 765 

Number of people who read only English newspapers = 123 + 206 + 325 = 654 

Number of people who read in both languages = 52 + 82 + 187 = 321 

=> Total number of people surveyed = 196 + 765 + 654 + 321 = 1936 

=> Ans – (D) 

English 

Instructions 

In the following questions, some part of the sentences have errors and some are correct. Find out which part of a sentence has an error. The number of that part is the answer. If a sentence is free from error, then your answer is (4) i.e., No error. 

Q. 76 

(A) Could you please give me 

(B) A postal address 

(C) of the Indian Embassy in New York 

(D) No error 

Answer: (B) 

 

Q. 77 

(A) Short stories and poems 

(B) of varying quality 

(C) appears in dailies and periodicals 

(D) No error 

Answer: (C) 

 

Q. 78 

(A) One of the 

(B) most dangerous disease 

(C) is AIDS 

(D) No error 

Answer: (B) 

Instructions 

In the following questions, sentence are given with blanks to be filled with an appropriate word(s). Four alternatives are suggested for each question. Choose the correct alternative out of the four. 

Q. 79 Throughout his career, his performance has fairly been _____ 

(A) consistence 

(B) consistent 

(C) consisting 

(D) constituted 

Answer: (B) 

 

Q. 80 I convey my thanks _____ the members of the club. 

(A) for 

(B) of 

(C) to 

(D) about 

Answer: (C) 

 

Q. 81 The government ____ on this issue. 

(A) is divided 

(B) are divided 

(C) is being divided 

(D) divided 

Answer: (A) 

 

Q. 82 The student is yet ___ his home task. 

(A) completion 

(B) compete 

(C) complete 

(D) continue 

Answer: (C) 

Instructions 

In the following questions, out of the four alternatives, choose the one which best expresses the meaning of the given word. 

Q. 83 Hard 

(A) difficult 

(B) simple 

(C) common 

(D) easy 

Answer: (A) 

 

Q. 84 Humorous 

(A) witty 

(B) innovative 

(C) fashionable 

(D) timid 

Answer: (A) 

 

Q. 85 Gather 

(A) scatter 

(B) disperse 

(C) congregate 

(D) separate 

Answer: (C) 

Instructions 

In the following questions, choose the word opposite in meaning to the given word. 

 

Q. 86 Slave 

(A) surf 

(B) landlord 

(C) master 

(D) tenant 

Answer: (C) 

 

Q. 87 Deep 

(A) shallow 

(B) hollow 

(C) steep 

(D) low 

Answer: (A) 

 

Q. 88 Egoist 

(A) spiritless 

(B) sulfless 

(C) senseless 

(D) soulless 

Answer: (B) 

Instructions 

In the following questions, four alternatives are given for the Idiom / Phrase printed in bold. Choose the alternative which best expresses the meaning of the Idiom / Phrase. 

Q. 89 I have told you time and again not to make this mistake. 

(A) always 

(B) often 

(C) sometimes 

(D) rarely 

Answer: (B) 

 

Q. 90 He handled the situation with an iron first. 

(A) strictly 

(B) leniently 

(C) softly 

(D) wayward 

Answer: (A) 

 

Q. 91 She is leaving the country for good. 

(A) for the time being 

(B) for good times 

(C) temporarily 

(D) permanently 

Answer: (D) 

Instructions 

In the following questions, a part of the sentence is printed in bold. Below are given alternatives to the bold part at (1), (2) and (3) which may improve the sentence. Choose the correct alternative. In case no improvement is needed your answer is (4) 

Q. 92 It has been raining since morning. 

(A) from 

(B) for 

(C) during 

(D) No improvement 

Answer: (D) 

 

Q. 93 I am neither a poet nor philosopher. 

(A) not philosopher 

(B) nor the philosopher 

(C) nor a philosopher 

(D) No Improvement 

Answer: (C) 

 

Q. 94 He was hung for murder 

(A) hang 

(B) hanged 

(C) hanging 

(D) No improvement 

Answer: (B) 

Instructions 

In the following questions, out of the following questions, out of the four alternatives, choose the one which can be substituted for the given words/sentences. 

 

Q. 95 An act of violence to take control of a plane 

(A) hold as hostage 

(B) abduct 

(C) hijack 

(D) kidnap 

Answer: (C) 

 

Q. 96 One who is all powerful 

(A) omnipotent 

(B) omniscient 

(C) absolute 

(D) almighty 

Answer: (A) 

 

Q. 97 That which can not be believed 

(A) inaudible 

(B) incredible 

(C) invincible 

(D) indivisible 

Answer: (B) 

Instructions 

In the following questions, groups of four words are given. In each group, one word is correctly spelt. Find the correctly spelt word. 

 

Q. 98 

(A) Elecution 

(B) Elocation 

(C) Elocution 

(D) Elocutiun 

Answer: (C) 

 

Q. 99 

(A) Juxttaposition 

(B) Justaposition 

(C) Jaxtaposition 

(D) Jaustaposition 

Answer: (A) 

 

Q. 100 

(A) Hazardous 

(B) Hazardos 

(C) Hazzardous 

(D) Hazardus 

Answer: (A) 

SSC GD 2011 Previous Year Paper

SSC GD 2011 

Quant 

Instructions 

For the following questions answer them individually 

Q. 1 A 270 meters long train running at the speed of 120 kmph crosses another train running in opposite direction at the speed of 80 kmph in 9 seconds. What is the length of the other train ? 

(A) 240 meters 

(B) 320 meters 

(C) 260 meters 

(D) 230 meters 

Answer: (D) 

 

Q. 2 Raviraj invested an amount of Rs.10,000 at compound interest rate of 10 p.e.p.a. For a period of three years. How much amount will Raviraj et after three years ? 

(A) Rs.13,210 

(B) Rs.13,310 

(C) Rs.12,100 

(D) Rs.11,000 

Answer: (B) 

 

Q. 3 Twice the square of a number is the cube of 18. The number is 

(A) 54 

(B) 108 

(C) 162 

(D) 324 

Answer: (A) 

 

Q. 4 The average salary of a group of 27 is Rs. 3,700. If the salary of one more person is added, the average is increased to Rs.3750. What is the salary of the new person ? 

(A) Rs.5010 

(B) Rs.5200 

(C) Rs.5100 

(D) Rs.5000 

Answer: (C) 

 

Q. 5 What should come in place of both the question marks (?) in the following equation. 

16/? = ?/42.25 

(A) 2.6 

(B) 260 

(C) .26 

(D) 26 

Answer: (D) 

 

Q. 6 An amount of money is to be distributed among P, Q and R in the ratio of 5:9:17 respectively. If the total of the shares of P and Q is Rs.7,000. What is R’s share in it 

(A) Rs.4,500 

(B) Rs.2,500 

(C) Rs.8,500 

(D) Rs.6,000 

Answer: (C) 

 

Q. 7 One-fourth of three-fifth of a number is 42. What is 40% of that number ? 

(A) 140 

(B) 116 

(C) 128 

(D) 112 

Answer: (D) 

 

Q. 8 By how much is 10% of 24.2 more than 10% of 24.02 ? 

(A) 1.8 

(B) 0.018 

(C) 0.18 

(D) 18 

Answer: (B) 

 

Q. 9 Ramesh bought a calculator with 20% discount on the tag price. He obtained 10% profit by selling it for Rs.440. What was the tag price ? 

(A) Rs.500 

(B) Rs.400 

(C) Rs.480 

(D) Rs.360 

Answer: (A) 

 

Q. 10 The sum of two numbers is 22 and their difference is 14. Find the product of numbers. 

(A) 72 

(B) 82 

(C) 62 

(D) 27 

Answer: (A) 

 

Q. 11 If m = 9 and n = ⅓m, then √(m)2 – (n)2 = ?

(A) 2

(B) 6

(C) 4

(D) 5

Answer: (B) 

 

Q. 12 The ratio between the ages of x and y at present is 3:4. Five years hence, the ratio of their ages will be 4:5; what is the present age of y in years ? 

(A) 15 

(B) 20 

(C) 25 

(D) 30 

Answer: (B) 

 

Q. 13 What would come in place of ($) mark in the following equation ? 

* 2 $ 20 ÷ 156 = 145 

(A)

(B)

(C)

(D)

Answer: (A) 

 

Q. 14 2⅕x2 = 2750, find the value of x ? 

(A) 25 

(B) 25

(C) 25

(D) 20 

Answer: (C) 

 

Q. 15 75×75−26×26/101 = ? 

(A) 59 

(B) 39 

(C) 29 

(D) 49 

Answer: (D) 

 

Q. 16 ∠ABC is an isosceles triangle and AB = AC = 2a unit BC = a unit, Draw AD ⊥ BC, and find the length of AD

(A) 15 a unit 

(B) 15/2 a unit 

(C) 17 a unit 

(D) 17/2 a unit 

Answer: (B) 

 

Q. 17 All sides of a quadrilateral ABCD touch a circle. If AB = 6 cm. BC = 7.5 cm. CD = 3 cm, then DA is 

(A) 3.5 cm 

(B) 4.5 cm 

(C) 2.5 cm 

(D) 1.5 cm 

Answer: (D) 

 

Q. 18 In a right angled triangle, the product of two sides is equal to half of the square of the third side i.e., hypotenuse. One of the acute angles must be 

(A) 60 

(B) 30 

(C) 45 

(D) 15 

Answer: (C) 

 

Q. 19 If two concentric circles are of radii 5 cm and 3 cm, then the length of the chord of the larger circle which touches the smaller circle is 

(A) 6 cm 

(B) 7 cm 

(C) 10 cm 

(D) 8 cm 

Answer: (D) 

 

Q. 20 Inside a square ABCD, △BEC is an equilateral triangle. If CE and BD interesect at O, then ∠BOC is equal to 

(A) 60 

(B) 75 

(C) 90 

(D) 120 

Answer: (B) 

 

Q. 21 A point D is taken from the side BC of a right angled triangle ABC, where AB is hypotenuse. Then, 

(A) AB2 + CD2 = BC2 + AD2 

(B) CD2 + BD2 = 2AD2 

(C) AB2 + AC2 = 2AD2 

(D) AB2 + AD2 = BD2 

Answer: (A) 

 

Q. 22 Let C be a point on a straight line AB. Circles are drawn with diameters AC and AB. Let P be any point on the circumference of the circle with diameter AB. If AP meets the other circle at Q, then 

(A) QC// PB 

(B) QC is never parallel to PB 

(C) QC = ½ PB 

(D) QC // PB and QC = ½ PB

Answer: (A) 

 

Q. 23 An isosceles triangle ABC is right angled at B.D is a point inside the triangle ABC. P and Q are the feet of the perpendiculars drawn from D on the sides AB and Ac respectively of △ABC. If AP = a cm, AQ = b cm and ∠BAD = 15 , sin = 75 

(A) 2b / √3a

(B) a/2b

(C) √3a /2b

(D) 2a / √3b

Answer: (C) 

 

Q. 24 Each interior angle of a regular octagon in radians is 

(A) π/4

(B) 3π/4

(C) 2π/3

(D) 1/3π

Answer: (B) 

 

Q. 25 Find the value of 30+30+…….

(A)

(B)

(C)

(D)

Answer: (C) 

Reasoning 

Instructions 

Select the related word/number from the given alternatives. 

Q. 26 Brain : Nerves : : Computer ? 

(A) Calculator 

(B) Keyboard 

(C) Mouse 

(D) CPU 

Answer: (D) 

Explanation: 

Nerves helps the brain to function in the same way CPU helps the computer to function. Hence, option (D) is the correct answer. 

Q. 27 Silkworm : Silk Saree : : Cobra 😕 

(A) Antidote 

(B) Poison 

(C) Death 

(D) Fear 

Answer: (B) 

Explanation: 

Silkworm produces silk whereas cobra produces poison. 

Hence, option (B) is the correct answer. 

Instructions 

Find the odd word from the given alternatives. 

Q. 28 

(A) Cover 

(B) Enclose 

(C) Bag 

(D) Annex 

Answer: (C) 

 

Q. 29 

a: Illusion 

b: Delusion 

c: Identification 

d: Hallucination 

(A) A

(B) B

(C)

(D) D

Answer: (C) 

Explanation: 

All the given words are synonyms except ‘Identification’ 

Hence, option (C) is the correct answer. 

Instructions 

For the following questions answer them individually 

Q. 30 Arrange the following words according to dictionary: 

1: Inadequate 

2: Institution 

3: Inhospitable 

4: Improvement 

(A) 4, 2, 3, 5, 1 

(B) 4, 1, 3, 5, 2 

(C) 4, 1, 5, 3, 2 

(D) 4, 1, 5, 2, 3 

Answer: (C) 

 

Q. 31 Identify the correct answer containing letters which will most appropriately fill in the blanks. 

a b a – a b – b – b a – 

(A) a, a, a, b 

(B) b, a, b, a 

(C) b, a, a, b 

(D) a, b, b, b 

Answer: (C) 

Explanation: 

The pattern followed here is, 

a b a b a b a b a b a b (repeating ‘ab’ for 6 times). Option (C) exactly fits in the blanks. Hence, option (C) is the correct answer. 

Instructions 

Select the missing letter/number from the given responses. 

Q. 32 

?, 187, 2057, 22627 

(A) 25 

(B) 27 

(C) 15 

(D) 17 

Answer: (D) 

Explanation: 

Every number is multiplied by 11. 

17 x 11 = 187, 

187 x 11 = 2057, 

2057 x 11 = 22627. 

Hence, option (D) is the correct answer. 

 

Q. 33 

C, F, I, L, ?, R, U, X 

(A) (A) 

(B)

(C)

(D)

Answer: (D) 

Explanation: 

The pattern followed here is, 

C + 2 = F, 

F + 2 = I, 

I + 2 = L, 

L + 2 = O……… 

Hence, option (D) is the correct answer. 

Instructions 

For the following questions answer them individually 

 

Q. 34 Nikhil was facing East. He walked 6 km forward and then after turning to his right walked 2 km. Again he turned to his right and walked 6 k. After this, he turned back. Which direction he was facing at that time ? 

(A) East 

(B) West 

(C) North 

(D) North-South 

Answer: (A) 

Explanation: 

Nikhil faces East finally after turning back. 

Hence, option (A) is the correct answer. 

 

Q. 35 Six boys are standing in such a manner that they form a circle facing the centre. Anand is to the left of Ravi. Shankar is in between Ajay and Vivek. Ishwar is between Anand and Ajay. Who is to the left of Vivek ? 

(A) Ravi 

(B) Ishwar 

(C) Ajay 

(D) Shankar 

Answer: (A) 

Explanation: 

Anand is to the left of Ravi. Ishwar is between Anand and Ajay. From this conditions arrangement will be, 

Shankar is in between Ajay and Vivek. Final arrangement will be, 

From the above arrangement, 

Ravi is to the left of vivek. 

Hence, option (A) is the correct answer. 

 

Q. 36 From the given alternatives, select the word which cannot be formed using the letters of the given word. 

CONSIDERATION 

(A) CONSIDER 

(B) CONCERN 

(C) NATION 

(D) RATION 

Answer: (B) 

Explanation: 

Except the word “CONCERN” other words can be formed using “CONSIDERATION” There is only one ‘C’ in CONSIDERATION. 

Hence, option (B) is the correct answer. 

 

Q. 37 If EARN is written as GCTP, how NEAR can be written in that code ? 

(A) CTGP 

(B) GPTC 

(C) PGCT 

(D) PCGT 

Answer: (C) 

Explanation: 

The pattern followed here is, 

E + 2 = G ; A + 2 = C ; R + 2 = T ; N + 2 = P i.e (n + 2) 

NEAR is coded as, 

N + 2 = P ; E + 2 = G ; A + 2 = C ; R + 2 = T 

NEAR – PGCT 

Hence, option (C) is the correct answer. 

 

Q. 38 If AMPLIFY is written as YFILPMA In a certain code, how would NATIONAL be written in that code ? 

(A) LANONATI 

(B) LANOITAN 

(C) LANTANIO 

(D) LANTION 

Answer: (B) 

Explanation: 

The positions of alphabets in the given word are reversed. 

Hence, the code for NATIONAL will be LANOITAN 

Hence, option (B) is the correct answer. 

 

Q. 39 The population of a developing country is increasing year by year. Find out the current year population, from the following information: 

Year 2004 2005 2006 2007 2008 2009 

Pop. in lakhs 30 60 120 210 230 ? 

(A) 390 

(B) 450 

(C) 480 

(D) 510 

Answer: (C) 

 

Q. 40 Gokul travelled 16 kms west ward, then he turned left and travelled 10 kms. Then he turned left and travelled 16 kms. How far was Gokul from the starting point ? 

(A) 16 kms 

(B) 26 kms 

(C) 10 kms 

(D) 6 kms 

Answer: (C) 

Explanation: 

From the given information, 

Gokul was 10 kms far from his starting point. 

Hence, option (C) is the correct answer. 

 

Q. 41 In a code language, the following code is used for the alphabets in a particular way: 

A B C D E H I J O P R S T 

Latex 

(A) LATCH 

(B) PATIO 

(C) PATCH 

(D) LATER 

Answer: (C) 

 

Q. 42 Refer to the information provided below 

‘M + N’ mens ‘M’ is father of ‘N’ 

‘M – N’ means ‘M’ is daughter of ‘N’ 

‘M X N’ means ‘M is son of N’ 

‘M ÷ N’ means ‘M is wife of ‘N’ 

How is D related to G in the expression D – F + G ? 

(A) Mother 

(B) Sister 

(C) Cannot be determined 

(D) Daughter 

Answer: (B) 

Explanation: 

(D) is the sister of G 

Hence, option (B) is the correct answer. 

 

Q. 43 A nuclear fuel disintegrates as shown with time 

9.10 am – 32 g 

9.35 am – 16 g 

10.00 am – 8 g 

10.25 am – 4 g 

And left at 11.40 am_____ 

(A) 2 g 

(B) 1 g 

(C) 500 mg 

(D) 0.5 mg 

Answer: (D) 

Explanation: 

The nuclear fuel disintegrates half of its value every 25 min. 

So, at 10:50 it becomes ‘2g’, at 11:15 it becomes ‘1g’ and at 11:40 it becomes 0.5g Hence, option (D) is the correct answer. 

 

Q. 44 Which of the answer figure is the mirror image of the given figure if the mirror is held at AB ? 

Answer: (D) 

 

Q. 45 In the given figures, triangle represents girls studying in a girls school, square represents boys studying in boys school and circle represents students studying in matriculation school. The portion which represent girls and boys studying in matriculation school is 

(A)

(B)

(C)

(D) D

Answer: (D) 

 

Q. 46 Which answer figure will complete the pattern in the question figure ? 

Answer: (A) 

 

Q. 47 A sheet of paper is folded in a particular manner, punched once and then unfolded. Punched, unfolded paper appears as, in the given figure. Find out the manner in which the paper was folded and punched by choosing the correct answer figure. 

Answer: (C) 

 

Q.48 A word is represented by only one set opf any one of the alternatives. The Sets of numbers, given in the alternatives are represented by two classes of alphabets as in two matrices given below. The columns and rows of Matrix I are numbered from 0 to 4 and that of Matrix II are numbered from 5 to 9. A letter from these matrices can be represented first by its row and next by its column, e.g. ‘F’ can be represented by 01, 13, etc., and \P\ can be represented by 66, 78, etc. Similarly, you have to identify the set for the word ‘MEN’. 

(A) 56, 32, 69 

(B) 55, 31, 95 

(C) 75, 00, 95 

(D) 76, 00, 88 

Answer: (C) 

Explanation: 

As per the given question, 

‘M’ can be represented by 56, 68, 75, 87, 99. 

Similarly, 

E – 00, 12, 24, 30, 31, 43. 

N – 57, 69, 76, 88, 95. 

“75, 00, 95” is one of the combinations of the word MEN 

Hence, option (C) is the correct answer. 

 

Q. 49 Select the missing number ?

2 2 8 

3 3 18 

4 4 ? 

(A) 30 

(B) 16 

(C)

(D) 32 

Answer: (D) 

Explanation: 

The pattern followed here is, 

Square of 1st digit is added to the square of 2nd digit. 

22 + 22 = 8 

32 + 32 = 18 

42 + 42 = 32 

Hence, option (D) is the correct answer. 

 

Q. 50 Select the missing number ?

6 7 21 

8 6 24 

12 9 ? 

(A) 54 

(B) 60 

(C) 18 

(D) 27 

Answer: (A) 

Explanation: 

The pattern followed here is, 

(6 x 7) / 2 = 21, 

(8 x 6) / 2 = 24, 

(12 x 9) / 2 = 54. 

Hence, option (A) is the correct answer. 

General Awareness 

Instructions 

For the following questions answer them individually 

Q. 51 As a result of 1857 Revolt, which ruler of native states of India was made prisoner ? 

(A) Begum of Avadh 

(B) Rani of Jhansi 

(C) Bahadur Shah 

(D) Raja of Vizianagaram 

Answer: (A) 

 

Q. 52 What was the main aim of home Rule Movement ? 

(A) To remove British rule 

(B) To turn out Britishera 

(C) To keep unity among Hindu-Muslims and fight for freedom 

(D) To attain self-government within the British Empire 

Answer: (D) 

 

Q. 53 When was the Non-violent Non-cooperation Movement started and by whom ? 

(A) In 1919 by Motilal Nehru 

(B) In 1919 by Mahatma Gandhi 

(C) In 1920 by Mahatma Gandhi 

(D) In 1920 by Lala Lajpat Rai 

Answer: (C) 

 

Q. 54 Who was the Viceroy of India at the time of formation of the Indian National congress ? 

(A) Lord Canning 

(B) Lord Dufferin 

(C) Lord Mayo 

(D) Lord Elgin 

Answer: (B) 

 

Q. 55 A person of which of the following blood groups can receive blood of any group ? 

(A) A

(B) AB 

(C)

(D)

Answer: (B) 

 

Q. 56 The percentage of carbon in case iron is:  

(A) 3 to 5 

(B) 0.1 to 0.25 

(C) 0.5 to 15 

(D) 6 to 8 

Answer: (A) 

 

Q. 57 Which gas is used as tear gas ? 

(A) Latex1 

(B) Latex2 

(C) Calcium sulphide (CaS) 

(D) Carbon sulphydry1 (CS) 

Answer: (D) 

 

Q. 58 The Indian Constitution is regarded as 

(A) Federal 

(B) Unitary 

(C) Parliamentary 

(D) Federal in form and unitary in spirit 

Answer: (D) 

Q. 59 Right to Property a 

(A) Fundamental Right 

(B) Directive Principle 

(C) Legal Right 

(D) Social Right 

Answer: (C) 

 

Q. 60 The President of India can declare Emergency if there is 

(A) War of threat of war 

(B) Failure of Constitutional machinery 

(C) Financial instability 

(D) Any of these three 

Answer: (D) 

 

Q. 61 In which year was the colour television introduced in India 

(A) 1979 

(B) 1980 

(C) 1981 

(D) 1982 

Answer: (D) 

 

Q. 62 Nepanagar in Madhya Pradesh is famous for 

(A) Textile 

(B) Newsprint paper 

(C) Hosiery 

(D) Vegetable oil 

Answer: (B) 

 

Q. 63 Which of the following is a direct tax ? 

(A) Excise 

(B) Sales Tax 

(C) Income tax 

(D) None 

Answer: (C) 

 

Q. 64 Paper currency first started in India in 

(A) 1862 

(B) 1542 

(C) 1601 

(D) 1680 

Answer: (A) 

 

Q. 65 How many banks were nationalized in 1969 ? 

(A) 16 

(B) 14 

(C) 15 

(D) 20 

Answer: (B) 

 

Q. 66 FICN stands for 

(A) Fair Indian Currency Note 

(B) Fake Indian Currency Note 

(C) Fresh Indian Currency Note 

(D) Forged Indian Currency Note 

Answer: (B) 

 

Q. 67 India’s first talkie film produced in 1931 was 

(A) Shakuntala 

(B) Alam Ara 

(C) Indra Sabha 

(D) Neel Kamal 

Answer: (B) 

 

Q. 68 Short-sightedness can be corrected by 

(A) convex lens 

(B) concave lens 

(C) novex-concave lens 

(D) concave-convex lens 

Answer: (B) 

 

Q. 69 The velocity of light was first measured by 

(A) Einstein 

(B) Newton 

(C) Roemer 

(D) Galileo 

Answer: (C) 

 

Q. 70 Which UN Agency has its H.Q. in Paris ? 

(A) UNICEF 

(B) ILO 

(C) UNESCO 

(D) FAO 

Answer: (C) 

 

Q. 71 The Indus and Brahmaputra rivers are examples of 

(A) Subsequent drainage 

(B) Super-imposed drainage 

(C) Consequent drainage 

(D) Antecedent drainage 

Answer: (B) 

 

Q. 72 ‘Terra rossa’ is a latin word which means 

(A) hot area 

(B) red terrain 

(C) lateritic region 

(D) region near to the poles 

Answer: (B) 

 

Q. 73 Which country has launched new virtual currency “petro” ? 

(A) South Africa 

(B) Venezuela 

(C) Indonesia 

(D) Japan 

Answer: (B) 

 

Q. 74 “Yakuts” are the nomadic herders of 

(A) Gobi 

(B) Sahara 

(C) Tundra 

(D) Kalahari 

Answer: (C) 

 

Q. 75 Beightan cup is associates with 

(A) Hoceky 

(B) Football 

(C) Cricket 

(D) Tennis 

Answer: (A) 

English 

Instructions 

Some parts of the sentences have errors and some have none. Find out which part of a sentence has an error, the appropriate letter (1, 2, 3). If a sentence is free from error, (4) is the Answer Sheet. 

Q. 76 

(A) A senior doctor 

(B) expressed concern 

(C) about physicians recommended the vaccine 

(D) No error 

Answer: (C) 

 

Q. 77 

(A) We have discussing 

(B) all the known mechanisms 

(C) of physical growth 

(D) No error 

Answer: (A) 

Instructions 

Sentence are given with blanks to be filled in with an appropriate word(s). Four alternatives are suggested for each question. Choose the correct alternative out of the four and indicate it in the Answer Sheet. 

Q. 78 If you had followed the rules, you………………disqualified. 

(A) will not be 

(B) would not be 

(C) will not have been 

(D) would not have been 

Answer: (D) 

 

Q. 79 The housewife ………… the cakes burning, and ran to switch off the oven. 

(A) smell 

(B) smells 

(C) smelt 

(D) smelling 

Answer: (C) 

 

Q. 80 …………. An old legend, King Rama lived in India. 

(A) In the event of 

(B) Due to 

(C) According to 

(D) In reference to 

Answer: (C) 

Instructions 

Out of the alternatives, choose the one which best expresses the meaning of the given word and mark it in the Answer Sheet. 

Q. 81 GENIAL 

(A) Cordial 

(B) Unselfish 

(C) Careful 

(D) Specific 

Answer: (A) 

 

Q. 82 ACCRUE 

(A) Accumulate 

(B) Accommodate 

(C) Grow 

(D) Suffice 

Answer: (A) 

 

Q. 83 LOQUACIOUS 

(A) Talkative 

(B) Slow 

(C) Content 

(D) Unclear 

Answer: (A) 

Instructions 

In questions choose the word opposite in meaning to the given word and mark it in the Answer Sheet. 

Q. 84 SYNTHETIC

(A) Natural 

(B) Plastic 

(C) Cosmetic 

(D) Apathetic 

Answer: (A) 

 

Q. 85 ACCORD 

(A) Disagreement 

(B) Welcome 

(C) Disrespect 

(D) Conformity 

Answer: (A) 

 

Q. 86 INFIRMITY 

(A) Employment 

(B) Indisposition 

(C) Strength 

(D) Weakness 

Answer: (C) 

Instructions 

In question, four alternatives are given for the idiom/phrase. Choose the alternatives which best expresses the meaning of the idiom/phrase and mark it in the Answer Sheet 

 

Q. 87 To be all at sea 

(A) a family voyage 

(B) lost and confused 

(C) in the middle of the ocean 

(D) a string of islands 

Answer: (B) 

 

Q. 88 To take to one’s heels 

(A) to walk slowly 

(B) to run away 

(C) to march forward 

(D) to hop and jump 

Answer: (B) 

 

Q. 89 To bite the dust 

(A) eat voraciously 

(B) have nothing to eat 

(C) eat roots 

(D) None of the above 

Answer: (D) 

Instructions 

A part of the sentence is underlined. Below are given alternatives to the underlined part at 1, 2, and 3 which may improve the sentence. Choose the correct alternative. In case no improvement is needed, your answer is 4. 

 

Q. 90 My friend lives in a nearby street whose name I have forgotten. 

(A) the name of which 

(B) which name 

(C) of which name 

(D) No improvement 

Answer: (D) 

 

Q. 91 He both won a medal and a scholarship 

(A) He won a medal and a scholarship both 

(B) Both he won a medal and a scholarship 

(C) He won both a medal and a scholarship 

(D) No improvement 

Answer: (C) 

 

Q. 92 He has for good left India. 

(A) He has left for good India 

(B) He has left India for good 

(C) Good he has left India 

(D) No improvement 

Answer: (B) 

Instructions 

Out of the four alternatives, choose the one which can be substituted for the given words/sentence. 

 

Q. 93 Pertaining to cattle 

(A) Canine 

(B) Feline 

(C) Bovine 

(D) Verminous 

Answer: (C) 

 

Q. 94 To look at someone in an angry or threatening way 

(A) Glower 

(B) Gnaw 

(C) Gnash 

(D) Grind 

Answer: (A) 

 

Q. 95 A post with little work but high salary 

(A) Director 

(B) Trustee 

(C) Sinecure 

(D) Ombudsman 

Answer: (C) 

Instructions 

For the following questions answer them individually 

 

Q. 96 An interview is important 

(A) only for the recruiting agency 

(B) only for the candidate 

(C) only for the public outside 

(D) both for the candidate and the recruiting agency 

Answer: (D) 

 

Q. 97 Surveys on interviews emerged with 

(A) flattering results 

(B) misgivings on them as a mode of selection 

(C) reasonable results 

(D) glowing tributes 

Answer: (B) 

Instructions 

In following Questions, a sentence has been given in Active/Passive voice out of the four alternatives suggested, select the one that best expresses the some sentence in passive/active voice and select your answer accordingly. 

 

Q. 98 She looks after the child 

(A) The child are looked after by her 

(B) The child were looked after by her 

(C) The child is looked after by her 

(D) None of these 

Answer: (C) 

 

Q. 99 Who stole your pen ? 

(A) Who was stole your pen ? 

(B) By whom is your pen stolen ? 

(C) By whom was your pen stolen ? 

(D) By whom did your pen steal ? 

Answer: (C) 

 

Q. 100 In the given Question, find the correctly spelt word. 

(A) Supersede 

(B) Superseed 

(C) supercede 

(D) supperssed 

Answer: (A) 

SSC GD 22 Feb 2019 Shift-III Previous Year Paper

SSC GD 22nd Feb 2019 Shift-III 

Reasoning 

Instructions 

For the following questions answer them individually 

Q. 1 Identify the odd pair from the following. 

(A) Tuberculosis : Heart 

(B) Jaundice : Liver 

(C) Rickets : Bones 

(D) Night Blindness : Eyes 

Answer: (A) 

Explanation: 

The disease in the first affect the body part in second, but tuberculosis affects the lungs, hence it is the odd one. 

=> Ans – (A) 

 

Q. 2 Read the given statements and conclusions carefully and decide which of the conclusions logically follow(s) from the statements. Statements: 

Some animals are birds. 

Cow is an animal. 

Conclusions: 

I. Some birds are animals. 

II. Cow is not a bird. 

(A) Neither conclusion I nor conclusion II follows. 

(B) Both conclusions I and II follow. 

(C) Only conclusion I follows. 

(D) Only conclusion II follows. 

Answer: (C) 

Explanation: 

The venn diagram for above statements is : 

Conclusions: 

I. Some birds are animals : follows 

II. Cow is not a bird : may or may not be true. 

Thus, only conclusion I follows. 

=> Ans – (C) 

 

Q. 3 Identify the option that belongs to the following group. 

Lamb, Calf, Tadpole 

(A) Amphibians 

(B) Puppy 

(C) Wild animals 

(D) Reptiles 

Answer: (B) 

 

Q. 4 Four persons (two men and two women) were sitting in a restaurant. They were each seated at the north, east, south and west of a table. No woman was facing the east. Persons sitting opposite to each other were not of the same gender. One man was facing the south. Which directions were the women facing? 

(A) North and west 

(B) South and west 

(C) East and west 

(D) North and south 

Answer: (A) 

 

Q. 5 If MARKET is coded as NCUOJZ, then RESPECT will be coded as ……….. 

(A) SGVTIJA

(B) SGVTIJZ 

(C) SGVTJIZ 

(D) SGVTJIA 

Answer: (D) 

Explanation: 

MARKET is coded as NCUOJZ 

Similarly, RESPECT : SGVTJIA 

=> Ans – (D) 

 

Q. 6 A square transparent paper after folding looks like as shown in the figure by solid lines. How will it look when unfolded? (Rotation is not allowed) 

Answer: (D) 

Explanation: 

When the figure will be unfolded, out of the two lines formed, clearly there will be a horizontal line in the middle, hence first and third options are eliminated. 

Of the remaining two, the second small vertical line will be formed below the horizontal line, thus last option best describes above figure. 

=> Ans – (D) 

 

Q. 7 Identify the odd one from the following. 

(A) Brother 

(B) Sister 

(C) Uncle 

(D) Friend 

Answer: (D) 

Explanation: 

All of brother, sister and uncle are blood relatives, hence friend is the odd one out. 

=> Ans – (D) 

 

Q. 8 Select the option that is related to the third number in the same way as the second number is related to the first number. 

1 : 121 :: 5 : ……. 

(A) 225 

(B) 25 

(C) 255 

(D) 125 

Answer: (A) 

Explanation: 

Expression = 1 : 121 :: 5 : ? 

x : (x + 10)2 

The pattern followed is = 

Eg :- (1 + 10)2 = (11)2 = 121 

Similarly, 

(5 + 10)2 = (15)2 = 225 

=> Ans – (A) 

 

Q. 9 Select the option in which the given figure is embedded. 

Answer: (C) 

 

Q. 10 Select the number that DOES NOT belong to the following group. 

1, 1, 1.5, 3, 7.5, 22.5, 78.75, 310 

(A) 22.5 

(B) 310 

(C) 78.75 

(D) 1.5 

Answer: (B) 

 

Q. 11 In the given Venn diagram, the rectangle represents students, the triangle represents teachers and the circle represents players. Study the diagram carefully and answer the question that follows. How many students are players but not teachers? 

(A)

(B)

(C)

(D)

Answer: (A) 

 

Q. 12 There is only one dictionary in a library which will be used by 6 students P, Q, R, S, T and U,one by one. S will use it immediately after Q and before P. T will use it immediately after R and before U. Q will use the dictionary after U. Who will use the dictionary in the end? 

(A)

(B)

(C)

(D)

Answer: (D) 

 

Q. 13 Identify the number pair that is different from the rest. 

(A) 45 – 5 

(B) 18 – 2 

(C) 72 – 9 

(D) 90 – 10 

Answer: (C) 

 

Q. 14 Select the option that most closely resembles the mirror image of the given word. 

TWIN 

(A) IIIWT 

(B) NIMT 

(C) NIWT 

(D) TWIN 

Answer: (A) 

 

Q. 15 If D = 3, G = 6, and OWL = 47, then FLOWER = ……….. 

(A) 89 

(B) 71 

(C) 73 

(D) 82 

Answer: (C) 

Explanation: 

Given : (D) = 3, G = 6 

Thus, the alphabets are represented by consecutive whole numbers : A=0, B=1, C=2, D=3, and so on. OWL = 14+22+11=47 

=> FLOWER = 5+11+14+22+4+17=73 

=> Ans – (C) 

 

Q. 16 Identify the number that will come in the following series. 

2500, 2400, 2200, 1900, 1500, …….. 

(A) 1000 

(B) 900 

(C) 1110 

(D) 1100 

Answer: (A) 

Explanation: 

The pattern followed is : 

2500 – 100 = 2400 

2400 – 200 = 2200 

2200 – 300 = 1900 

1900 – 400 = 1500 

1500 – 500 = 1000 

=> Ans – (A) 

 

Q. 17 Select the correct option to complete the following pattern. 

Answer: (C) 

 

Q. 18 Select the term that will come next in the following series. 

Z2A, X4D, V6G, T8J, R10M, ………… 

(A) Q12Q 

(B) Q12P 

(C) P12Q 

(D) P12P 

Answer: (D) 

Explanation: 

Series : Z2A, X4D, V6G, T8J, R10M, 

The numbers in the middle are consecutive even numbers. 

1st letter : Z (-2 letters) = X (-2 letters) = V (-2 letters) = T (-2 letters) = R (-2 letters) = P 3rd letter : (A) (+3 letters) = (D) (+3 letters) = G (+3 letters) = J (+3 letters) = M (+3 letters) = P Thus, missing term = P12P 

=> Ans – (D) 

 

Q. 19 Select the option that is analogous to the given pair. 

Nation : President 

(A) State : Minister 

(B) State : Chief Minister 

(C) State : Governor 

(D) State : Chief Secretary 

Answer: (C) 

Explanation: 

President and Governor are the nominal heads of the nation and the state respectively. Thus, State : Governor 

=> Ans – (C) 

 

Q. 20 Which two signs need to be interchanged to make the given equation correct? 

5 × 5 + 5 − 5 ÷ 5 = 5 

(A) × and − 

(B) + and × 

(C) + and − 

(D) ÷  and − 

Answer: (A) 

 

Q. 21 Six friends P, Q, R, S, T and U are watching a cricket match sitting in a stadium.P is sitting at one extreme end. T is sitting between P and U.R is sitting to the immediate left of U and immediate right of Q. Who is sitting at the extreme right end? 

(A)

(B)

(C)

(D)

Answer: (C) 

Explanation: 

P is sitting at one extreme end, the two possible cases are (assuming all are facing north). 

i) : P __ __ __ __ __ 

ii) : __ __ __ __ __ P 

T is sitting between P and U.R is sitting to the immediate left of U and immediate right of Q, 

i) : P T U __ __ __ 

ii) : __ __ __ U T P 

Thus, first case is not possible, arrangement : S Q R U T P 

P is sitting at extreme right end. 

=> Ans – (C) 

 

Q. 22 Read the given statements and conclusions carefully and decide which of the conclusions logically follow(s) from the statements. Statements: 

The national norm is 150 beds per thousand population,but in this state, 200 beds per thousand are available in the hospitals. 

Conclusions: 

I. The state’s norm is appropriate. 

II. The state’s health system is taking adequate care in this regard. 

(A) Neither conclusion I nor conclusion II follows. 

(B) Only conclusion I follows. 

(C) Only conclusion II follows. 

(D) Both conclusions I and II follow. 

Answer: (C) 

Explanation: 

The given statements indicate that for every thousand population in a state, there must be 150 beds in the hospitals, and if there are more beds, we cannot conclude that it is appropriate, also, there are 200 beds in this state, thus we can definitely conclude that the state’s health system is taking adequate care in this regard. 

Thus, only conclusion II follows. 

=> Ans – (C) 

 

Q. 23 In the following number series, two numbers have been put with in brackets. Study the series carefully and select the correct alternative with respect to the series. 

0, 1, 4, 9, (15), 25, 36, 49, (64), 81 

(A) Both the bracketed numbers are incorrect. 

(B) Both the bracketed numbers are correct. 

(C) The first bracketed number(from the left) is incorrect and the second is correct. 

(D) The first bracketed number(from the left) is correct and the second is incorrect. 

Answer: (C) 

Explanation: 

Clearly, the above series is a combination of square of consecutive whole numbers, but the first bracket number, i.e. 15 is incorrect as it 

is not a perfect square, while the second bracket number 64 is correct, as 

(8)2 = 64 

Thus, the first bracketed number(from the left) is incorrect and the second is correct. 

=> Ans – (C) 

 

Q. 24 Identify the odd one from the following. 

(A) HL 

(B) KN 

(C) CG 

(D) UY 

Answer: (B) 

Explanation: 

(A) : H (+4 letters) = L 

(B) : K (+3 letters) = N 

(C) : (C) (+4 letters) = G 

(D) : U (+4 letters) = Y 

=> Ans – (B) 

 

Q. 25 Select the combination that will come next in the following series. 

Answer: (B) 

General knowledge 

Instructions 

For the following questions answer them individually 

Q. 26 What was the brand name of the first electric powered washing machine? 

(A) Thor 

(B) Images 

(C) Icon 

(D) Goliath 

Answer: (A) 

 

Q. 27 When was the Aadhaar (Targeted Delivery of Financial and Other Subsidies, Benefits and Services) Act passed? 

(A) 2015 

(B) 2017 

(C) 2016 

(D) 2014 

Answer: (C) 

 

Q. 28 What does Individual Vulnerability refer to in Economics? 

(A) The threat to Intellectual property 

(B) The threat of Exploitation 

(C) The threat to health 

(D) The threat of poverty 

Answer: (D) 

 

Q. 29 In the WTO’s Agreement on Agriculture (AoA), what do the terms Green Box, Blue Box and AMS(Amber Box) stand for? 

(A) Interest 

(B) Subsidies 

(C) Bonus 

(D) Taxes 

Answer: (B) 

 

Q. 30 Which of the following animals hibernates? 

(A) Tiger 

(B) Bat 

(C) Jackal 

(D) Deer 

Answer: (B) 

 

Q. 31 Rungrado May Stadium, the largest football stadium in the world, is located in which country? 

(A) China 

(B) Singapore 

(C) North Korea 

(D) Spain 

Answer: (C) 

 

Q. 32 What is the highest peak of Pir Panjal Range? 

(A) Indrasan 

(B) Kabru 

(C) Nanda Devi 

(D) Kamet 

Answer: (A) 

 

Q. 33 Who is the Padma Bhushan awardee who guided ISRO to blast its heaviest ever rocket GSLV Mark-III, paving the road to manned space missions? 

(A) Fali Nariman 

(B) Manohar Parrikar 

(C) K. Radhakrishnan 

(D) Nandan Nilekani 

Answer: (C) 

 

Q. 34 Which country is the largest producer of mangoes as of 2018? 

(A) Brazil 

(B) Sri Lanka 

(C) India 

(D) China 

Answer: (C) 

 

Q. 35 Who invaded India in 326 BC after crossing the river Indus, advancing towards Taxila? 

(A) Eduardo Barbosa 

(B) Nicolini 

(C) Marco Polo 

(D) Alexander 

Answer: (D) 

 

Q. 36 ‘Paika’ is a traditional dance form associated with which Indian state? 

(A) Uttarakhand 

(B) Jharkhand 

(C) Sikkim 

(D) Madhya Pradesh 

Answer: (B) 

 

Q. 37 Which of the following diseases is caused by bats? 

(A) Dengue 

(B) Zika 

(C) Ebola 

(D) Chikungunya 

Answer: (C) 

 

Q. 38 Which of the following options is an ancient literary source that sheds light on the ancient period of Indian history? 

(A) Puranas 

(B) Bhagwat Gita 

(C) Upanishads 

(D) Sama Veda 

Answer: (A) 

 

Q. 39 In which country is the highest mountain peak of Africa Kilimanzaro located? 

(A) Kenya 

(B) Zambia 

(C) Ghana 

(D) Tanzania 

Answer: (D) 

 

Q. 40 Chaukhandi Stupa is a Buddhist monument located in which Indian 

state? 

(A) Punjab 

(B) Uttar Pradesh 

(C) Sikkim 

(D) West Bengal 

Answer: (B) 

 

Q. 41 ‘Jyotipunj’ is a book written by: 

(A) Narendra Modi 

(B) Dhirubhai Ambani 

(C) VS. Naipaul 

(D) Munshi Premchand 

Answer: (A) 

 

Q. 42 A deficiency of Vitamin A causes: 

(A) Anaemia 

(B) Goitre 

(C) Night blindness 

(D) Rickets 

Answer: (C) 

 

Q. 43 Which acids are present in most grapes? 

(A) Hydrochloric and sulphuric acid 

(B) Nitric and sulphuric 

(C) Tartaric and malic acid 

(D) Acetic acid and nitric acid 

Answer: (C) 

 

Q. 44 In which of the following Indian state was the game of polo born? 

(A) Assam 

(B) West Bengal 

(C) Arunachal Pradesh 

(D) Manipur 

Answer: (D) 

 

Q. 45 The Shanti Swaroop Bhatnagar Award is given in India for contribution to the field of: 

(A) sports 

(B) social work 

(C) science and technology 

(D) literature 

Answer: (C) 

 

Q. 46 In which city was the East India Association formed in the year 1866? 

(A) Kolkata 

(B) Singapore 

(C) London 

(D) Paris 

Answer: (C) 

 

Q. 47 In which amendment of the Constitution was the voting age reduced from 21 to 18 years? 

(A) 56th 

(B) 70th 

(C) 65th 

(D) 61st 

Answer: (D) 

 

Q. 48 Federalism is a system of government in which the powers is …….. between a central authority and various constituent units of the country. 

(A) Divided 

(B) Incited 

(C) Derided 

(D) Indicted 

Answer: (A) 

 

Q. 49 ‘Teeyan’is a festival celebrated in which state of India? 

(A) Gujarat 

(B) Jammu and Kashmir 

(C) Rajasthan 

(D) Punjab 

Answer: (D) 

 

Q. 50 What does Buffer stock mean? 

(A) (A) barrier on stock creation when surplusis available 

(B) (A) reserve of commodities used to offset deficit 

(C) (A) focus on production of one specific variety when there is a deficit 

(D) (A) preventive measure to stop excessive distribution 

Answer: (B) 

Quant 

Instructions 

For the following questions answer them individually 

Q. 51 Two trains are coming from opposite side at 40 km/hrs and 50 km/hrs from the different places 360 km apart. After how many hours two trains will meet to each other? 

(A) 4 hrs 

(B) 6 hrs 

(C) 5 hrs 

(D) 3 hrs 

Answer: (A) 

 

Q. 52 The greatest number that will divide 146, 248 and 611 leaving remainders 2,8 and 11 respectively is: 

(A) 47 

(B) 144 

(C) 24 

(D) 612 

Answer: (C) 

Explanation: 

The greatest number that will divide 146, 248 and 611 leaving remainders 2,8 and 11 respectively 

= H.C.F. of (146-2), (248-8), (611-11) 

= H.C.F. (144,240,600) = 24 

=> Ans – (C) 

 

Q. 53 Study the Table Properly and answer by interpreting the data. 

The table given the percentage of result by four different classes in the five different year of examination: 

In which class the average performance is least in all the five years? 

(A) Class-XII 

(B) Class-XI 

(C) Class-IX 

(D) Class-X 

Answer: (A) 

Explanation: 

Average performance is least in all the five years for class 

(A) : Class-XII = 75 + 58 + 33 + 35 + 38 = 239 [MIN] 

(B) : Class-XI = 85 + 62 + 32 + 38 + 32 = 249 

(C) : Class-IX = 90 + 70 + 65 + 50 + 55 = 330 

(D) : Class-X = 80 + 65 + 35 + 48 + 45 = 273 

=> Ans – (A) 

 

Q. 54 The value of (6 × 15) ÷ (2 × 3) − 22 + 3  is: 

(A) 12 

(B) 14 

(C)

(D)

Answer: (B) 

 

Q. 55 In a board examination, 75% of the total examinees passed. If the number of failures is 625, the total number of examinees is: 

(A) 3800 

(B) 2400 

(C) 3600 

(D) 2500 

Answer: (D) 

 

Q. 56 Study the Table Properly and answer by interpreting the data. 

The table given the percentage of result by four different classes in the five different year of examination: 

What is the average percentage of marks in Class-IX in the year 2012 to 2016? 

(A) 65% 

(B) 72% 

(C) 66% 

(D) 62% 

Answer: (C) 

 

Q. 57 A mixture contains milk and water in the ratio of 4 : 3. When 5 liters of water is added to this mixture, the ratio of milk and water becomes 6 : 7. The quantity of milk in the mixture is: 

(A) 12 liters 

(B) 5 liters 

(C) 15 liters 

(D) 10 liters 

Answer: (A) 

 

Q. 58 A dining set having a marked price of ₹4000 is sold at a discount of 8%, thereby earning a profit of 10%. The cost price of the dining set is: 

(A) ₹3345.45 

(B) ₹3786.78 

(C) ₹3568.80 

(D) ₹3496.50 

Answer: (A) 

 

Q. 59 An amount of ₹ 4600 has been distributed between x, y & z in the ratio of ½ : ⅔ : ¾. The share of y is: 

(A) 1600 

(B) 2000 

(C) 1200 

(D) 1800 

Answer: (A) 

 

Q. 60 The ratio between the speeds of two trains is 3 : 4. The second train runs 800 km in 5 hrs, and speed of first train is: 

(A) 120 km/hrs 

(B) 140 km/hrs 

(C) 180 km/hrs 

(D) 160 km/hrs 

Answer: (A) 

 

Q. 61 In a cricket team, the average age of 11 players is 22 years. When the age of team coach is included to it, the average age increases by 1. The age of coach of the team is: 

(A) 33 years 

(B) 34 years 

(C) 43 years 

(D) 53 years 

Answer: (B) 

 

Q. 62 Sheela buys 3 pens for ₹24 and sold at 2 for ₹18 and 1 for ₹9. The profit or loss percent is: 

(A) Loss 12.5% 

(B) Profit 12% 

(C) Loss 12% 

(D) Profit 12.5% 

Answer: (D) 

 

Q. 63 Sweta can paint a wall in 30 hours, while Ansul can paint it in 40 hours. If they paint it together and get payment of ₹ 14,000. What is Ansul’s share? 

(A) ₹6000/- 

(B) ₹12000/- 

(C) ₹10000/- 

(D) ₹8000/- 

Answer: (A) 

 

Q. 64 What will be the compound interest on ₹6000 at 3% per annum for 2 years compounded annually? 

(A) ₹365.40 

(B) ₹653.20 

(C) ₹372.50 

(D) ₹725.10 

Answer: (A) 

 

Q. 65 The scores done by a bats man in a cricket series are 12, 25, 38, 56, 64, 86. What will be the median? 

(A) 45 

(B) 56 

(C) 47 

(D) 38 

Answer: (C) 

 

Q. 66 A trader sold a machine for ₹18,800 and lost 6%. The cost price of the machine is: 

(A) ₹22000 

(B) ₹24000 

(C) ₹20000 

(D) ₹18000 

Answer: (C) 

 

Q. 67 The ratio of the area of two triangles is 2 : 3 and ratio of their height is 3 : 2. The ratio of their bases is: 

(A) 3:2 

(B) 4:9 

(C) 9:4 

(D) 2:3 

Answer: (B) 

 

Q. 68 Study the Table Properly and answer by interpreting the data. 

The table given the percentage of result by four different classes in the five different year of examination: 

In Which year the average performance is maximum in all the four classes? 

(A) 2016 

(B) 2015 

(C) 2013 

(D) 2012 

Answer: (D) 

 

Q. 69 The value of 62 − 5 of (18 − 14) + 5 × 7 is equal to: 

(A) 18 

(B) 28 

(C) 15 

(D) 77 

Answer: (D) 

 

Q. 70 If the radius of the circle is decreased by 10%. The percentage decreased in its areas is : 

(A) 90% 

(B) 110% 

(C) 19% 

(D) 91% 

Answer: (C) 

 

Q. 71 Simple interest on a certain sum is 16/25 of the total sum. If rate percent and time are same, what will be the rate of interest? 

(A) 6.8% 

(B) 5% 

(C) 12% 

(D) 8% 

Answer: (D) 

 

Q. 72 The average of five consecutive numbers is 15. The highest of these number is: 

(A) 12 

(B) 19 

(C) 17 

(D) 15 

Answer: (C) 

Explanation: 

Average of five consecutive numbers = 15 

=> Middle number = 15 

Thus, the consecutive numbers are = 13,14,15,16,17 

Thus, highest number = 17 

=> Ans – (C) 

 

Q. 73 Sanjya and Saumya together can finish a work in 9 days. Sanjya can do the same job on his own in 15 days. The same work Saumya can do by himself in: 

(A) 14 days 

(B) 15½ days 

(C) 22½ days 

(D) 22 days 

Answer: (C) 

 

Q. 74 The base of the parallelogram is twice of its height and its area is 288cm2. The base of the parallelogram is: 

(A) 12 cm 

(B) 24 cm 

(C) 36 cm 

(D) 28 cm 

Answer: (B) 

 

Q. 75 The mean proportional between 36 and 121 is equal to: 

(A) 72 

(B) 85 

(C) 66 

(D) 157 

Answer: (C) 

English 

Instructions 

For the following questions answer them individually 

 

Q. 76 From the given options, identify the segment in the sentence which contains the grammatical error. Pragati Maidan is the famous international exhibition center at Delhi for trade fairs. 

(A) for trade fairs 

(B) Pragati Maidan is 

(C) at Delhi 

(D) the famous 

Answer: (C) 

 

Q. 77 Select the synonym of the given word. 

PROBABLE 

(A) Tangible 

(B) Substitute 

(C) Likely 

(D) Similar 

Answer: (C) 

 

Q. 78 From the given options, identify the segment in the sentence which contains the grammatical error. Japan will be a country well known for its culture, technology and cuisines like sushi. 

(A) will be a country 

(B) well known for 

(C) cuisines like 

(D) its culture 

Answer: (A) 

 

Q. 79 Select the most appropriate option to substitute the underlined segment in the given sentence. If there is no need to substitute it, select No improvement. 

He work in a shop which makes handicraft goods. 

(A) works in a shop which make 

(B) works in a shop which makes 

(C) No improvement 

(D) working in a shop which makes 

Answer: (B) 

 

Q. 80 From the given options, identify the segment in the sentence which contains the grammatical error. The country also needs large doses of infrastructure additions — roads or highways, power plants, distribution networks, airports and ports. 

(A) airports and ports 

(B) also needs 

(C) roads or highways 

(D) The country 

Answer: (C) 

 

Q. 81 Select the word which means the same as the group of words given. 

One that has life 

(A) Mortal 

(B) Animate 

(C) Inanimate 

(D) Conscious 

Answer: (B) 

Instructions 

In the following passage some words have been deleted. Fill in the blanks with the help of the alternatives given. 

Passage: 

Bird-watching, as it is popularly called, is a peaceful …………(1) . Itis not entirely without occasional physical thrills and even …………(2) The elephant ridden jungles in South India, for instance. I found myself in uncomfortably disturbing …………(3) . Actually, there is little danger from a wild elephant unless it happens to be a female with a small ………….(4) along with her. What such situations demand is …………(5) and not valour. 

Q. 82 Select the most appropriate option that will fill in the blank number 1. 

(A) job 

(B) service 

(C) activity 

(D) work 

Answer: (C) 

 

Q. 83 Select the most appropriate option that will fill in the blank number 2. 

(A) conditions 

(B) hazards 

(C) pleasures 

(D) circumstances 

Answer: (B) 

 

Q. 84 Select the most appropriate option that will fill in the blank number 3. 

(A) zones 

(B) atmospheres 

(C) situations 

(D) locations 

Answer: (C) 

 

Q. 85 Select the most appropriate option that will fill in the blank number 4. 

(A) cub 

(B) calf 

(C) pup 

(D) kitten 

Answer: (B) 

 

Q. 86 Select the most appropriate option that will fill in the blank number 5. 

(A) loyalty 

(B) honesty 

(C) forgetfulness 

(D) discretion 

Answer: (D) 

Instructions 

For the following questions answer them individually 

 

Q. 87 Select the correctly spelt word. 

(A) Accessaries 

(B) Accessories 

(C) Acessories 

(D) Accesories 

Answer: (B) 

 

Q. 88 Select the most appropriate word to fill in the blank. 

Pollution of rivers and seas destroys ………. life. 

(A) Aerial 

(B) Marine 

(C) Desert 

(D) Terrestrial 

Answer: (B) 

 

Q. 89 Select the most appropriate option to fill in the blank. 

Their wages were very low, so the workers went on a ……….. 

(A) strike 

(B) leave 

(C) strife 

(D) conflict 

Answer: (A) 

 

Q. 90 Select the most appropriate ANTONYM of the given word. 

COARSE 

(A) Fine 

(B) Texture 

(C) Rough 

(D) Glossy 

Answer: (A) 

 

Q. 91 Select the option that means the same as the given idiom. 

Run of the mill 

(A) Ordinary 

(B) Rare 

(C) Exceptional 

(D) Uncommon 

Answer: (A) 

 

Q. 92 Select the most appropriate option to fill in the blank. 

Ten million people watch this television ………. every week. 

(A) incident 

(B) event 

(C) programme 

(D) media 

Answer: (C) 

 

Q. 93 Select the synonym of the given word. 

LETHARGY 

(A) Delight 

(B) Thrill 

(C) Wisdom 

(D) Fatigue 

Answer: (D) 

 

Q. 94 Select the most appropriate word to fill in the blank. 

People have used indigenous plants for ……….. of ailments since ancient times. 

(A) treatment 

(B) exclusion 

(C) adulteration 

(D) pollution 

Answer: (A) 

 

Q. 95 Select the option that means the same as the given idiom. 

Man of letters 

(A) A student 

(B) A scholar 

(C) One who writes letters 

(D) One who can read letters 

Answer: (B) 

 

Q. 96 Select the word which means the same as the group of words given. 

Those that feed only on plants 

(A) Omnivorous 

(B) Predatory 

(C) Amphibian 

(D) Herbivorous 

Answer: (D) 

 

Q. 97 Select the correctly spelt word. 

(A) Greivance 

(B) Grievanse 

(C) Grievence 

(D) Grievance 

Answer: (D) 

 

Q. 98 Select the most appropriate option to substitute the underlined segment in the given sentence. If there is no need to substitute it, select No improvement. 

it’s getting dark, can you switch the lights please? 

(A) No improvement 

(B) can you switch on the lights please? 

(C) can you switch ofthe lights please? 

(D) can you on the lights please? 

Answer: (B) 

 

Q. 99 Select the antonym of the given word. 

WHOLESALE 

(A) Discount 

(B) Retail 

(C) Concession 

(D) Rebate 

Answer: (B) 

 

Q. 100 Select the most appropriate option to substitute the underlined segment in the given sentence. If there is no need to substitute it, select No improvement. 

If he remains absent from the college, he will have to pay a fine. 

(A) If he is remain absent from 

(B) No improvement 

(C) If he is remaining absent from 

(D) If he will remains absent from 

Answer: (B) 

SSC GD 21 Feb 2019 Shift-III Previous Year Paper

SSC GD 21st Feb 2019 Shift-III 

Reasoning 

Instructions 

For the following questions answer them individually 

Q. 1 If A = 2 and D = 5, and SEA = 28, then COUNTRY = ……….. 

(A) 123 

(B) 132 

(C) 112 

(D) 121 

Answer: (A) 

Explanation: 

If A = 2 and D = 5, and SEA = 20+6+2=28 

4 + 16 + 22 + 15 + 21 + 19 + 26 = 123 

Then, COUNTRY = 

=> Ans – (A) 

 

Q. 2 Read the given statements and conclusions carefully and decide which of the conclusions logically follow(s) from the statements. Statements: 

1) All students in my class are active. 

2) Rohini is not active. 

Conclusions: 

I. Rohini is not a student of my class. 

II. Rohini must be active. 

(A) Both conclusions I and II follow. 

(B) Only conclusion I follows. 

(C) Only conclusion II follows. 

(D) Neither conclusion I nor II follows. 

Answer: (B) 

Explanation: 

The venn diagram for above statements is : 

Conclusions: 

I. Rohini is not a student of my class = true 

II. Rohini must be active. = false 

Thus, only conclusion I follows. 

=> Ans – (B) 

 

Q. 3 Choose the word from the given options that is similar to the given words and hence belongs to the same group. 

Lira, Dinar, Rupee 

(A) Lisbon 

(B) Cairo 

(C) Rial 

(D) Oslo 

Answer: (C) 

Explanation: 

Lira, Dinar and Rupee are all currencies of different countries, while in the options, only Rial is a currency of Saudi Arabia, rest are cities. 

=> Ans – (C) 

 

Q. 4 Select the correct option to complete the following pattern. 

Answer: (D) 

Explanation: 

If we complete the above figure, then the missing pattern is represented by the red colour. 

=> Ans – (D) 

 

Q. 5 In the following number series, two numbers have been put within brackets. Study the series carefully and select the correct alternative with respect to the series. 

0, 7, 26, 63, (124), 215, 342, (512) 

(A) The first bracketed number(from the left) is correct and the second is incorrect. 

(B) The first bracketed number (from the left) is incorrect and the second is correct. 

(C) Both the bracketed numbers are incorrect. 

(D) Both the bracketed numbers are correct. 

Answer: (A) 

 

Q. 6 Select the option that most closely resembles the mirror image of the given word when the mirror is placed vertically. 

Answer: (C) 

Explanation: 

When the mirror is placed vertically, the object at right will appear at left position and vice-versa. Thus, the letter ‘U’ at left will appear at right and since it’s symmetrical, its shape will not be changed also, thus third is the correct option. 

=> Ans – (C) 

 

Q. 7 Select the figure that will come next in the following series. 

Answer: (B) 

Explanation: 

In the three figures given, the arrow has pointed to the right, top and left respectively, hence going by the symmetry, the only position left to face is the bottom one as shown in image in the second option. 

=> Ans – (B) 

 

Q. 8 Select the number that DOES NOT belong to the following group. 

0, 5, 12, 20, 32, 45 

(A) 45 

(B) 32 

(C) 20 

(D) 12 

Answer: (C) 

 

Q. 9 In a private school, there were five teachers. Abi and Geetha were teaching French and English. Harini and Geetha were teaching English and Science. Diva and Abi were teaching Maths and French. Hema and Geetha were teaching Social Science and French. Which subject is taught by maximum number of teachers? 

(A) Science 

(B) Maths 

(C) French 

(D) English 

Answer: (C) 

Explanation: 

Teachers who are teaching each subject are : 

Science : Geetha, Harini 

Maths : Divya, Abi 

French : Abi, Geetha, Divya, Hema [max] 

English : Abi, Geetha, Harini 

=> Ans – (C) 

 

Q. 10 L, M, N and O are to be seated in a row. However, N and O cannot be together. Moreover, M cannot be in the third place from the left. L and M are sitting beside each other. Considering the seating from left to right which of the following must be true? 

(A) N is at the third place. 

(B) L is at the third place. 

(C) N is at the first place. 

(D) O is at the third place. 

Answer: (B) 

 

Q. 11 Select the Venn diagram that best represents the given set of classes. 

Animals, Reptiles, Snakes 

Answer: (B) 

 

Q. 12 Four numbers have been given out of which three are alike in some manner, while one is different. Choose the odd one. 

(A) 250 

(B) 95 

(C) 159 

(D) 150 

Answer: (C) 

Explanation: 

Apart from 159, all numbers are divisible by 5, hence it is the odd one. 

=> Ans – (C) 

 

Q. 13 Given is a statement, followed by four conclusions. Read the statement carefully and decide which of the conclusions logically follow(s) from the given statement. 

Statement: 

This book can help because all good books help. 

Conclusions: 

I. This is not a good book. 

II. This is a good book. 

III. No good book helps. 

IV. Some good books help. 

(A) Only conclusion I follows. 

(B) All the conclusions follow. 

(C) Only conclusion II follows. 

(D) Both conclusions III and IV follow. 

Answer: (C) 

Explanation: 

The given statement indicates that the given book can help as all good books help, which can definitely conclude that this is a good book. 

The last two conclusions are not true as they are contradictory to the given statement. 

Thus, only conclusion II follows. 

=> Ans – (C) 

 

Q. 14 Five friends are sitting in a circle, all facing the centre. Mohan is to the left of Raju. Vinith is to the right of Arjun and between Arjun and Naveen. Who is between Raju and Naveen? 

(A) Arjun and Vinith 

(B) Vinith 

(C) Arjun 

(D) Mohan 

Answer: (D) 

 

Q. 15 Four words have been given out of which three are alike in some manner, while one is different. Choose the odd one. 

(A) Necklace 

(B) Bracelet 

(C) Jewellery 

(D) Anklet 

Answer: (C) 

Explanation: 

Necklace, Bracelet and Anklet are all types of jewellery, hence jewellery is the odd one. 

=> Ans – (C) 

 

Q. 16 A square transparent sheet with a pattern is given. How will the pattern appear when the transparent sheet is folded along the dotted line? 

Answer: (D) 

 

Q. 17 Which two signs need to be interchanged to make the given equation correct? 

3 ÷ 2 − 4 × 5 + 5 = 1 

(A) + and ÷ 

(B) × and − 

(C) ÷ and × 

(D) + and − 

Answer: (A) 

 

Q. 18 Select the option in which the given figure is embedded. 

Answer: (C) 

 

Q. 19 Select the option that is related to the third term in the same way as the second term is related to the first term. 

Angle : Radian :: Mass : ………. 

(A) Kilogram 

(B) Litre 

(C) Degree 

(D) Height 

Answer: (A) 

Explanation: 

Second is the unit of first, angle is measured in radian, similarly unit of mass is kilogram.

=> Ans – (A) 

 

Q. 20 Select the correct option that will fill in the blank and complete the series. 

5, 22, 39, 56, ……? 

(A) 89 

(B) 76 

(C) 73 

(D) 87 

Answer: (C) 

Explanation: 

17 is added in all the terms 

5 + 17 = 22 

22 + 17 = 39 

39 + 17 = 56 

56 + 17 = 73 

=> Ans – (C) 

 

Q. 21 Identify the odd one from the following. 

(A) dFh 

(B) gHj 

(C) uWy 

(D) rTv 

Answer: (B) 

Explanation: 

(A) : d (+2 letters) = F (+2 letters) = h 

(B) : g (+1 letter) = H (+2 letters) = j 

(C) : u (+2 letters) = W (+2 letters) = y 

(D) : r (+2 letters) = T (+2 letters) = v 

=> Ans – (B) 

 

Q. 22 If CENTRAL is coded as BFMUQBK, then FRIENDS will be coded as ………… 

(A) ESHEMER 

(B) ESHFMER 

(C) ESHENER 

(D) ESHFNER 

Answer: (B) 

 

Q. 23 Four pairs of words have been given, out of which three are alike in some manner, while one is different. Choose out the odd one. 

(A) Chef : Knife 

(B) Saw : Carpenter 

(C) Sculptor : Chisel 

(D) Doctor : Stethoscope 

Answer: (B) 

Explanation: 

In all the given pairs, except second, second item is used by the first person, while in second reverse is true, hence it is the odd one.

 => Ans – (B) 

 

Q. 24 Select the option that is related to the third number in the same way as the second number is related to the first number. 

7 : 50 :: 13 : ? 

(A) 160 

(B) 144 

(C) 170 

(D) 169 

Answer: (C) 

Explanation: 

Expression = 7 : 50 :: 13 : ? 

x : x2 + 1 

The pattern followed is = 

Eg :- 72 + 1 = 50 

Similarly, 

(13)2 + 1 = 169 + 1 = 170 

=> Ans – (C) 

 

Q. 25 Select the term that will come next in the following series. 

YCL, MQZ, ZDM, NRA, ……, OSB

(A) AEN 

(B) BFQ 

(C) ADM 

(D) BEM 

Answer: (A) 

Explanation: 

Expression : YCL, MQZ, ZDM, NRA, ……, OSB 

The pattern followed in alternate terms is that each letter of the terms is replaced by next succeeding alphabet. 

Odd terms : YCL (+1) = ZDM (+1) = AEN 

Even terms : MQZ (+1) = NR(A) (+1) = OSB

Thus, missing term : AEN 

=> Ans – (A) 

General knowledge 

Instructions 

For the following questions answer them individually 

Q. 26 Which of the following statements about ‘Biological Magnification’ is FALSE? 

(A) Its maximum concentration is at the top level of the food chain. 

(B) Its maximum concentration is at the bottom level of the food chain. 

(C) It differs depending on the level of ecosystem. 

(D) The chemicals enter the food chain through absorption from water or soil. 

Answer: (B) 

 

Q. 27 The first Mughal emperor, Babur came to India from: 

(A) Syria 

(B) Greece 

(C)Ferghana 

(D) Persia 

Answer: (C) 

 

Q. 28 Banks were nationalised in India through an ordinance passed in the year: 

(A) 1976 

(B) 1963 

(C) 1969 

(D) 1954 

Answer: (C) 

 

Q. 29 Who presented the first Union Budget of Independent India? 

(A) NV Gadgil 

(B) Maulana Azad 

(C) CD Deshmukh 

(D) RK Shanmukham Chetty 

Answer: (D) 

 

Q. 30 ……… is the oldest mountain range in the Indian subcontinent. 

(A) Himalaya 

(B) Vindhya 

(C) Aravali 

(D) Satpura 

Answer: (C) 

 

Q. 31 ‘Sevasadan’, ‘Karmabhoomi’ and ‘Gaban’ are some of the masterpieces of which Hindi writer? 

(A) Bhisham Sahni 

(B) Premchand 

(C) Harishchandra 

(D) Ramdhari Singh ‘Dinkar’ 

Answer: (B) 

 

Q. 32 As of January 2019, the Chief Minister of Chhattisgarh is: 

(A) Shiv Dahariya 

(B) Bhupesh Baghel 

(C) Kamal Nath 

(D) Kawasi Lakhma 

Answer: (B) 

 

Q. 33 Which of the following diseases is caused by bacteria? 

(A) Rabies 

(B) Chicken Pox 

(C) Small Pox 

(D) Tuberculosis 

Answer: (D) 

 

Q. 34 Which of the following is NOT an amphibian? 

(A) Frog 

(B) Tortoise 

(C) Salamander 

(D) Toad 

Answer: (B) 

 

Q. 35 Life on earth began in the oceans almost ………. years ago. 

(A) 1 billion 

(B) 100 million 

(C) 500 million 

(D) 3.5 billion 

Answer: (D) 

 

Q. 36 Which chemical is widely used to unclog drains and sinks? 

(A) Sodium Chloride 

(B) Nitrogen Hydroxide 

(C) Hydrogen Peroxide 

(D) Sodium Hydroxide 

Answer: (D) 

 

Q. 37 The concept of five – year plans in the Constitution of India is borrowed from ……….. 

(A) Russia 

(B) England 

(C) The United States 

(D) Germany 

Answer: (A) 

 

Q. 38 The Fiscal Year in India is considered between: 

(A) 1 April – 31 March 

(B) 1 January – 31 December 

(C) 1 July – 30 June 

(D) 31 March – April 

Answer: (A) 

 

Q. 39 Which state hosts the annual Surajkund Crafts fair? 

(A) Rajasthan 

(B) Uttar Pradesh 

(C) Punjab 

(D) Haryana 

Answer: (D) 

 

Q. 40 Which of the following statements with respect to Article 22 of the Constitution and Criminal Law on fundamental rights of an arrested person is NOT correct? 

(A) The Right to be informed atthe time ofarrest of the offence for which the person is being arrested 

(B) A boy under 15 years of age and women cannot be called to the police station only for questioning 

(C) The Right to be presented before a magistrate within 24 hours ofarrest 

(D) Confessions made in police custody can be used as evidence against the accused 

Answer: (D) 

 

Q. 41 Which team has won the Ranji Trophy maximum number of times? 

(A) Saurashtra 

(B) Tamil Nadu 

(C) Mumbai 

(D) Vidarbha 

Answer: (C) 

 

Q. 42 Which country ranks first in the ‘Per Capita’ Summer Olympic medals tally? 

(A) Netherlands 

(B) Finland 

(C) Japan 

(D) Hungary 

Answer: (B) 

 

Q. 43 The Brahmaputra river is called ‘Tsangpo’ in Tibet. What is it called in Bangladesh? 

(A) Meghna 

(B) Padma 

(C) Karnaphuli 

(D) Jamuna 

Answer: (D) 

 

Q. 44 Chandragupta Maurya established the Mauryan Empire by defeating the: 

(A) Nandas 

(B) Chalukyas 

(C) Pallavas 

(D) Shungas 

Answer: (A) 

 

Q. 45 Victoria Memorial is located in: 

(A) Lahore 

(B) Mumbai 

(C) Kolkata 

(D) New Delhi 

Answer: (C) 

 

Q. 46 ‘Gidda’ is a dance form of which state? 

(A) Punjab 

(B) Assam 

(C) West Bengal 

(D) Odisha 

Answer: (A) 

 

Q. 47 Who was the first Indian woman president of Indian National Congress? 

(A) Vijaya Lakshmi Pandit 

(B) Sarojini Naidu 

(C) Sucheta Kriplani 

(D) Aruna Asaf Ali 

Answer: (B) 

 

Q. 48 Which of the following statements is not a good argument in favour of a democracy? 

(A) Democracies are more accountable to the people 

(B) Democracies have better resolutions of conflicts 

(C) People feel free and equal in a democracy 

(D) Democracies are more prosperous than others 

Answer: (D) 

 

Q. 49 Which of the following statements about sound is FALSE? 

(A) The audible range of an average human is 20 Hz — 20 kHz. 

(B) The speed of sound remains uniform irrespective of the temperature or medium. 

(C) Sound travels as a longitudinal wave. 

(D) Sound cannot travel in a vacuum. 

Answer: (B) 

 

Q. 50 The Dadasaheb Phalke Award announced in 2018 was awarded to: 

(A) Late Yash Chopra 

(B) Late Rajesh Khanna 

(C) Late Shashi Kapoor 

(D) Late Vinod Khanna 

Answer: (D) 

Quant 

Instructions 

For the following questions answer them individually 

Q. 51 If the interest on money be 1 paisa per rupee per month, what is the rate per cent per annum? 

(A) 12% 

(B) 15% 

(C) 10% 

 (D) 10½ % 

Answer: (A) 

 

Q. 52 The median of the data 28, 31, 42, 37, 26, 34, 18, 23 is: 

(A) 30 

(B) 29.5 

(C) 31.5 

(D) 28 

Answer: (B) 

 

Q. 53 Study the following graph which shows the production (in thousands)of different items, and answer these questions. 

The ratio between the number of pants produced by the company in the year 2014, 2015 and 2016 respectively is: 

(A) 3 : 4 : 6 

(B) 5 : 10 : 7 

(C) 1 : 2 : 3 

(D) 3 : 4 : 5 

Answer: (B) 

 

Q. 54 The area of an equilateral triangle is 173 cm2 then of the perimeter the equilateral triangle (use 3 = 1.73) is: 

(A) 40 cm 

(B) 60 cm 

(C) 100 cm 

(D) 20 cm 

Answer: (B) 

 

Q. 55 A pen drive was sold at a price after giving two successive discounts of 30% and 20%. If the selling price of the item was ₹ 560, then what was the marked price of the pen drive? 

(A) ₹1000 

(B) ₹840 

(C) ₹1140 

(D) ₹1280 

Answer: (A) 

 

Q. 56 A discount of 30% on the marked price of a shirt enables a man to purchase a tie also, which costs him ₹ 210. How much did the man pay for the shirt? 

(A) ₹540 

(B) ₹490 

(C) ₹630 

(D) ₹700 

Answer: (B) 

 

Q. 57 The cost price of 120 bags is ₹ 15 each. 40 of them were sold for ₹18 each. At what price should the remaining be sold in order to get an overall profit of ₹ 4 per bag? 

(A) ₹19.5 

(B) ₹22.5 

(C) ₹16 

(D) ₹20 

Answer: (A) 

 

Q. 58 Study the following graph which shows the production (in thousands) of different items, and answer these questions.

The total number of all products produced by the company in the year 2014 and 2016 together is: 

(A) 105700 

(B) 108500 

(C) 1075 

(D) 10750 

Answer: (B) 

Explanation: 

Total number of all products produced by the company in the year 2014 and 2016 together : 

[(10 + 7.5 + 15) + (25 + 30 + 21)] × 1000 = 

108500 

=> Ans – (B) 

 

Q. 59 Seven persons have 20, 30, 40, 50, 60, 70 and 80 chocolates with them. If each of them is given 5 chocolates additionally, what will be the average number of chocolates with them? 

(A) 50 

(B) 55 

(C) 45 

(D) 60 

Answer: (B) 

 

Q. 60 A beats B by 125 m in a kilometer race. Find B’s speed if A’s speed is 16 m/sec.: 

(A) 18 m/sec 

(B) 14 m/sec 

(C) 7 m/sec 

(D) 21 m/sec 

Answer: (B) 

 

Q. 61 The ratio of the present ages of Meera and Sheela is 9 : 5. After 8 years Sheela would reach the present age of Meera. What is the present age (in years) of Sheela? 

(A) 18 

(B) 10 

(C)

(D)

Answer: (B) 

 

Q. 62 The difference between simple interest and compound interest on ₹ 2500 at 4% per annum for two years is (in ₹): 

(A) ₹40 

(B) ₹4 

(C) ₹45 

(D) ₹14 

Answer: (B) 

 

Q. 63 Study the following graph which shows the production (in thousands)of different items, and answer these questions.

The difference between the total number of Shoes and Shirts produced by the company in the year 2016 and the number of Pants produced by the company in the year 2014 is: 

(A) 40000 

(B) 35000 

(C) 32000 

(D) 38000 

Answer: (A) 

 

Q. 64 Suresh sells a pen at ₹ 120 for 33⅓% profit. At what price should he sell it to gain 40%? 

(A) ₹160 

(B) ₹120 

(C) ₹140 

(D) ₹126 

Answer: (D) 

 

Q. 65 If 12 men or 20 women can do a work in 54 days, then in how many days can 9 men and 12 women together do the work? 

(A) 38 

(B) 32 

(C) 40 

(D) 35 

Answer: (C) 

 

Q. 66 What should come in place of the question mark (?) in the following question? 

1 − [5 − {2 + (−5 + 6 − 2)2}] =? 

(A)

(B)

(C) -2 

(D) -4 

Answer: (D) 

 

Q. 67 A started running towards north direction at 6:00 a.m. B also started running towards north direction at 10:00 a.m. At what time will they meet if the speeds of A and are in the ratio 3 : 7 and they started from the same point? 

(A) 1 p.m. 

(B) 2 p.m. 

(C) 1:30 p.m. 

(D) 12:30 p.m. 

Answer: (A) 

 

Q. 68 The area of the base of a right circular cone is 1408/7 cm2 and its height is 6 cm. Taking π = 22/7 , the curved surface area of the cone is: 

(A) 1670/7 cm2

(B) 1067/7 cm2

(C) 1760/7 cm2

(D) 7160/7 cm2

Answer: (C) 

 

Q. 69 The perimeter(in metres) of a semicircle is numerically equal to its area (in square meters). The length of its diameter is (Take ) 

Answer: (A) 

 

Q. 70 There are two numbers a2 and 9. Their mean proportion is b, then (a2 + b ) : (a2 – b2) = ?

(A) 5 : 4 

(B) 3 : 4 

(C) 3 : 2 

(D) 1 : 1 

Answer: (A) 

 

Q. 71 The average number of chocolates with six children is six. If another child joined them the average number of chocolates with the seven children will become 7. What is the number of chocolates with the seventh child? 

(A) 14 

(B) 11 

(C) 12 

(D) 13 

Answer: (D) 

 

Q. 72 A and B working together can complete a job in 30 days. The ratio of their efficiencies is 3 : 2. In how many days can the faster person complete the job? 

(A) 50 

(B) 30 

(C) 40 

(D) 60 

Answer: (A) 

 

Q. 73 What should come in place of the question mark (?) in the following question? 

2 − [6 − {3 + (−4 + 5 + 1) × 8} + 12] =? 

(A)

(B)

(C)

(D)

Answer: (C) 

 

Q. 74 Find the least square number which is divisible by 4, 8, 2, 6 and 12? 

(A) 36 

(B) 24 

(C) 48 

(D) 144 

Answer: (D) 

 

Q. 75 In a class of 80 students, 60% passed in Economics and only 20% passed in History. The number of students who neither passed in Economics nor in History are: 

(A) 20 

(B) 12 

(C) 24 

(D) 16 

Answer: (D) 

English 

Instructions 

For the following questions answer them individually 

Q. 76 Select the most appropriate option to substitute the underlined segment in the given sentence.If there is no need to substitute it, select No improvement. 

Even if you did not know where I was, you can always have telephoned me to ask. 

(A) you could always have telephoned me 

(B) No improvement 

(C) you will always have telephoned me 

(D) you should always have telephoned me 

Answer: (A) 

Instructions 

In the following passage some words have been deleted. Fill in the blanks with the help of the alternatives given. 

Passage: 

Women have ………….(1) been held in high esteem in our country. During the Vedic period they …………(2) important positions in social and religious ………..(3) Without women, religious ceremonies were ……………(4) incomplete. Our mythological and philosophical writings bear witness to the fact that the tradition of having high regard for women was long ……………..(5) 

 

Q. 77 Select the most appropriate option that will fill in the blank number 1. 

(A) never 

(B) seldom 

(C) always 

(D) hardly 

Answer: (C) 

 

Q. 78 Select the most appropriate option that will fill in the blank number 2. 

(A) occupied 

(B) developed 

(C) demanded 

(D) offered 

Answer: (A) 

 

Q. 79 Select the most appropriate option that will fill in the blank number 3. 

(A) books 

(B) literature 

(C) life 

(D) thoughts 

Answer: (C) 

 

Q. 80 Select the most appropriate option that will fill in the blank number 4. 

(A) accepted 

(B) believed 

(C) performed 

(D) considered 

Answer: (D) 

 

Q. 81 Select the most appropriate option that will fill in the blank number 5. 

(A) awaited 

(B) maintained 

(C) valued 

(D) discontinued 

Answer: (B) 

Instructions 

For the following questions answer them individually 

 

Q. 82 From the given options, identify the segment in the sentence which contains the grammatical error. When questioned about their laziness, almost all the boys gave the same excuse, ‘We all are tired.’ 

(A) When questioned 

(B) almost all the boys 

(C) ‘We all are tired.’ 

(D) about their laziness 

Answer: (C) 

Instructions 

Select the most appropriate option to fill in the blank. 

 

Q. 83 Macbeth is the tragedy of a man who was ………. with great qualities. 

(A) endowed 

(B) conferred 

(C) empowered 

(D) advantaged 

Answer: (A) 

 

Q. 84 All the respondents should express their ………. views in this questionnaire. 

(A) candid 

(B) confident 

(C) convincing 

(D) contrary 

Answer: (A) 

Instructions 

For the following questions answer them individually 

 

Q. 85 Select the option that means the same as the given idiom. 

To split hairs 

(A) To occupy one’s time in dressing 

(B) To stop the argument in between 

(C) To get angry about someone’s fashion 

(D) To engage in discussing irrelevant details 

Answer: (D) 

 

Q. 86 Select the most appropriate word to fill in the blank. 

The government should provide attractive tax ……….. for the salaried class. 

(A) structures 

(B) documents 

(C) suggestions 

(D) bids 

Answer: (A) 

 

Q. 87 Select the most appropriate option to substitute the underlined segment in the given sentence. If there is no need to substitute it, select No improvement. 

In spite of his illness, the man run a few yards before withdrawing from the race. 

(A) ran a few yards before withdrew from 

(B) ran a few yards before withdrawing from 

(C) No improvement 

(D) running a few yards before withdrawing from 

Answer: (B) 

 

Q. 88 Select the word which means the same as the group of words given. 

Organisms that live on another 

(A) Bacteria 

(B) Microbes 

(C) Viruses 

(D) Parasites 

Answer: (D) 

 

Q. 89 From the given options, identify the segment in the sentence which contains the grammatical error. The person at the counter when filling up my details asked me how many languages did I know. 

(A) did I know 

(B) at the counter 

(C) filling up my details 

(D) how many languages 

Answer: (A) 

 

Q. 90 Select the option that means the same as the given idiom. 

To read between the lines 

(A) To ask someone to read every line 

(B) To understand more than actual words 

(C) To look for the meaning of every word 

(D) To understand the dictionary meanings 

Answer: (B) 

 

Q. 91 Select the correctly spelt word. 

(A) Scisors 

(B) Scissors 

(C) Scissars 

(D) Sissors 

Answer: (B) 

 

Q. 92 Select the correctly spelt word. 

(A) Allteration 

(B) Altaration 

(C) Alterashion 

(D) Alteration 

Answer: (D) 

 

Q. 93 From the given options, identify the segment in the sentence which contains the grammatical error. Soon I was informed byreliable sources that their parents had no any responsibilities towards the family. 

(A) towards the family 

(B) by reliable sources 

(C) Soon I was informed 

(D) no any responsibilities 

Answer: (D) 

 

Q. 94 Select the synonym of the given word. 

CONFLUENCE 

(A) Junction 

(B) Flow 

(C) Diversion 

(D) Stream 

Answer: (A) 

 

Q. 95 Select the antonym of the given word. 

Genuine 

(A) True 

(B) False 

(C) Frank 

(D) Actual 

Answer: (B) 

 

Q. 96 Select the most appropriate option to substitute the underlined segment in the given sentence. If there is no need to substitute it, select No improvement. 

The City Development Council resolved at a recent meeting to take steps to controlling pollution immediately. 

(A) to take steps to control! pollution immediately 

(B) No improvement 

(C) to take steps of control pollution immediately 

(D) to take steps for control pollution immediately 

Answer: (A) 

 

Q. 97 Select the word which means the same as the group of words given. 

A short stay at a place 

(A) Voyage 

(B) Lodge 

(C) Sojourn 

(D) Trip 

Answer: (C) 

 

Q. 98 Select the synonym of the given word. 

PSEUDONYM 

(A) Family name 

(B) Alias 

(C) Namesake 

(D) First name 

Answer: (B) 

 

Q. 99 Select the most appropriate word to fill in the blank. 

The Director has still not given his ……….. to conduct the survey of natural resources. 

(A) compliance 

(B) projection 

(C) consent 

(D) request 

Answer: (C) 

 

Q. 100 Select the antonym of the given word. 

Interruption 

(A) Disruption 

(B) Hinderance 

(C) Continuity 

(D) Temporary 

Answer: (C) 

×

Hello!

Click one of our representatives below to chat on WhatsApp or send us an email to info@vidhyarthidarpan.com

×